You are on page 1of 124

INTRODUCTION TO ACCOUNTING

MCQ BANK

COMPILED BY: SIR TAHA POPATIA

WhatsApp: 0345 3086312


INDEX
S.No Chapter Name Questions Answers
1 Accounting Fundamentals 01 98
2 Books of Prime Entry 10 99
3 Ledgers and Trial Balance 18 101
4 Accruals and Prepayments 27 103
5 Bad and Doubtful Debts 36 105
6 Accounting for Depreciation 46 108
7 IAS-2 Inventories 55 111
8 Accounting for Manufacturing 64 113
9 Preparation of Financial Statements 74 115
10 Bank Reconciliation Statement 88 119
Chapter 1 ACCOUNTING FUNDAMENTALS
Question#1
Which statement regarding bookkeeping is correct?

A. Book-keeping is same as accounting B. Book-keeping is concerned with


summarizing, analyzing and reporting
these transactions in the financial
statements.
C. Book-keeping is concerned with only D. Both B and C
recording financial transactions of the
business

Question#2
Which is not the part of accounting cycle?
A. Transactions B. Books of prime entry and financial
statements
C. Ledgers and Trial balance D. None

Question#3
Any transaction that has a monetary impact on the business’ accounts is?
A. Cash transaction B. Financial transaction
C. Credit transaction D. Both A and C

Question#4
A business bought machinery for production.
Which type of transaction is this?
A. Capital transaction B. Revenue transaction
C. One-off transaction D. Both A and C

Question#5
Business overhauled machinery used in production.
Which type of expenditure is this?
A. Revenue expenditure B. Normal expenditure
C. Capital expenditure D. Ongoing transaction

Question#6
Printing press incurred minor repairs to its printing machine which is already used in printing.
What is the nature of this expenditure?
A. Revenue expenditure B. Normal expenditure
C. Capital expenditure D. Ongoing transaction

Question#7
Which statement regarding accounting is correct?

A. Book-keeping is same as accounting B. Accounting is concerned with

Prepared by Maaz Imtiaz


1
summarizing, analyzing and reporting
these transactions in the financial
statements.
C. Accounting is concerned with only D. Both B and C
recording financial transactions of the
business

Question#8
Financial statements are reports of an entity which provide necessary information to its ________ for
their decision making needs.
A. Shareholders B. Directors
C. Government D. Stakeholders

Question#9
Accounting equation is?
A. Assets = Liabilities + Equity B. Assets = Liabilities - Equity
C. Assets + Liabilities = Equity D. None

Question#10
Which is not an accounting equation?
A. Assets = Liabilities + Equity B. Assets = Liabilities – Equity
C. Assets – Liabilities = Equity D. Both Band C

Question#11
Which of the following is correct?
A. Revenue – Cost of sales = Gross profit B. Revenue + Cost of sales = Gross profit
C. Revenue – Gross profit = Cost of sales D. Both A and C

Question#12
Which of the following is correct?
A. Net profit = Gross profit – Expenses B. Net profit – Expenses = Gross Profit
C. Gross profit – Expenses = Cost of sales D. None

Question#13
How is gross profit calculated?
A. Sales – expenses B. Sales – sales return – cost of goods sold
C. Sales – purchase returns D. Sales – sales return – closing stock

Question#14
Which best describe the capital of the business?
A. Items which the business owns B. Items which business bought for long
term use
C. Cash invested by owner of business D. Items that turn into cash in near future

Question#15
Maaz enterprises income statement showed a gross profit but a loss for the year.
Why this is so?

Prepared by Maaz Imtiaz


2
A. Sales exceeds purchases B. Expenses exceeds cost of sales
C. Sales exceeds cost of sales D. Expenses exceeds gross profit

Question#16
Which best describe the capital of the business?
A. Items which the business owns B. Items which business bought for long
term use
C. Cash invested by owner of business D. Items that turn into cash in near future

Question#17
Which would not appear in balance sheet?
A. Fixed assets B. Gross profit
C. Trade receivables D. Accrued rent expense

Question#18
How the business of beauty parlor can be classified?
A. Service business B. Exchange business
C. Production business D. Manufacturing business

Question#19
Plant and building can be classified into?
A. Assets B. Noncurrent liabilities
C. Noncurrent assets D. Current liabilities

Question#20
Trade payables can be classified into?
A. Assets B. Noncurrent liabilities
C. Non-current assets D. Current liablities

Question#21
Long term bank loan can be classified into?
A. Assets B. Non current liablities
C. Non current assets D. Current liablities

Question#22
Samiullah textiles has incurred following costs for the year ended 30 June 2021:
Extention in factory building : Rs2 Mn
Repairs to factory building : Rs 0.6 Mn
Overhaul to machinery that increased production capacity : Rs1.9 Mn
What is the amount of capital expenditure ?
A. Rs3.9 Mn B. Rs4.5 Mn
C. Rs0.6 Mn D. Rs2.5 Mn

Question#23

Prepared by Maaz Imtiaz


3
Samiullah textiles has incurred following costs for the year ended 30 June 2021:
Extention in factory building : Rs2 Mn
Repairs to factory building : Rs 0.6 Mn
Overhaul to machinery that increased production capacity : Rs1.9 Mn
What is the amount of revenue expenditure ?
A. Rs3.9 Mn B. Rs4.5 Mn
C. Rs0.6 Mn D. Rs2.5 Mn

Question#24
Samiullah textiles has incurred following costs for the year ended 30 June 2021:
Extention in factory building : Rs2 Mn
Repairs to factory building : Rs 0.6 Mn
Overhaul to machinery that increased production capacity : Rs1.9 Mn
Profit for the year was Rs10.6 Mn .
What will be the profit after incorporating above adjustments ?
A. Rs8.6 Mn B. Rs6.7 Mn
C. Rs8.7 Mn D. Rs10 Mn

Question#25
Yousuf Flour Mills has incurred following costs for the year ended 30 June 2021:
Rs in Mn
Cost of factory building – opening 18
Cost of factory vehicle – opening 9.9
Construction of new building – during the 5
year
Repairs to building – during the year 1.6
Repairs to factory vehicle – during the 0.2
year
What is the cost of building at year end?
A. Rs10.1Mn B. Rs23 Mn
C. Rs24.6Mn D. Rs9.9Mn

Question#26
Yousuf Flour Mills has incurred following costs for the year ended 30 June 2021:
Rs in Mn
Cost of factory building – opening 18
Cost of factory vehicle – opening 9.9
Construction of new building – during the 5
year
Repairs to building – during the year 1.6
Repairs to factory vehicle – during the 0.2
year
What is the cost of factory vehicle at year end?
A. Rs10.1Mn B. Rs23 Mn

Prepared by Maaz Imtiaz


4
C. Rs24.6Mn D. Rs9.9Mn

Question#27
Owner bought factory vehicle from his personal cash.
How owner’s expenditure is classified?
A. Capital B. Payable
C. Liability D. Asset

Question#28
When a business owner takes goods away for personal use is
A. Drawings B. Sales
C. Capital D. Purchase returns

Question#29
Which is of the following is not current asset.
A. Plant B. Trade recievables
C. Prepaid rent D. Interest recievable on loan

Question#30
Which is of the following is not non-current asset.
A. Trade recievables B. Plant
C. Building D. Factory vehicles

Question#31
What is an asset acoording to IASB ?
A. A present economic resource controlled B. A resource which decreases liablity
by the entity as a result of past events ,
from which future economic benefits
flows to the entity .
C. A present economic resource controlled D. A resource which increases equity .
by the entity as a result of past events.

Question#32
If asset of business is Rs 3.2Mn and liablities is Rs1.2Mn . Then what would be the amount of equity ?
A. Rs2Mn B. Rs4.4Mn
C. Rs3.2Mn D. Rs4Mn

Question#33
At 31 October 2020 Yosuf trial balance included the following balances:
Rs
Machinery 15,840

Inventory 8,990

Trade receivables 7,050

Trade payables 8,040

Prepared by Maaz Imtiaz


5
Bank overdraft 6,670

Cash at bank 8,950


What is the amount of current assets?
A. 40,830 B. 9,150
C. 24,990 D. 22,930

Question#34
At 31 October 2020 Yosuf trial balance included the following balances:
Rs
Machinery 15,840

Inventory 8,990

Trade receivables 7,050

Trade payables 8,040

Bank overdraft 6,670

Cash at bank 8,950


What is the amount of non-current assets?
A. 24,830 B. 15,840
C. 31,880 D. 22,930

Question#35
Which of the following require most detailed financial information from financial statements?
A. Management B. Shareholders
C. Employees D. Government

Question#36
Responsibility to prepare financial statements lies with ?
A. Management B. Lenders
C. Shareholders D. Tax agencies

Question#37
Purchase of factory vehicle for cash
A. Increase Assets B. Total assets remains unchanged
C. Decrease cash D. All of the above

Question#38
Sales made for cash will result in ;
A. Increase in cash B. Increase in cash and decrease in equity
C. Increase in sales D. Both A and C

Question#39
What transaction is presented by the entries: debit Payables, credit Bank?

Prepared by Maaz Imtiaz


6
A. Made payment to supplier from bank B. Receive cheque from bank
C. Purchase of goods on credit D. Sale of goods for credit

Question#40
As a rule , a credit entry usually represents increase in;
A. Income and liabilities B. Asset and liabilities
C. Income and expenses D. Liabilities and expenses

Question#41
Kashif enterprises sells Rs. 500,000 worth of goods to a customer, the customer pays Rs. 90,000 in
cash immediately and will pay the remaining Rs. 410,000 in 60 days’ time.
What is the double entry to record the purchase in the customer’s accounting records?
A. Debit purchases Rs. 500,000; credit cash B. Dr. purchases Rs. 500,000; Cr. payables
Rs. 500,000 Rs. 410,000; Cr. cash Rs. 90,000
C. Dr. purchases Rs. 500,000; Dr. payables D. Dr. cash Rs. 90,000; Cr. payables Rs.
Rs. 410,000; Cr. cash Rs. 90,000 410,000; Cr. purchases Rs. 500,000

Question#42
Chemical industry has purchased machinery on credit. Which of the accounts are affected by the
transactions?

A. Purchases B. Cash
C. Trade payables D. Machinery

Question#43
A company buy a vehicle amounting Rs2Mn for business use and made payment from cheque .
What will the double entry ?
A. Vehicle Dr , Bank Cr B. Purchases Dr , Cash Cr
C. Vehicle Dr , Cash Cr D. Purchases Dr , Bank Cr

Question#44
The double entry of rent expense paid through cash is ;
A. Equipment Dr , Cash Cr B. Rent Expense Dr , Cash Cr
C. Equipment Dr , Bank Cr D. Rent Dr , Expense Cr

Question#45
Mr Akram sold goods worth Rs200,000 on credit ; What will be the double entry?
A. Sales Dr , Trade recievables Cr B. Trade payables Dr , Profit Cr
C. Trade recievables Dr , Sales Cr D. Trade payables Dr , Income Cr

Question#46
A business had following balances for its plant and machinery .
Rs in Mn
Opening 2.6
Additions during the year 0.6

Prepared by Maaz Imtiaz


7
Disposals during the year 0.9
Closing ?
What is the amount of closing balance at year end?
A. Rs2.3Mn B. Rs4.1Mn
C. Rs2.9Mn D. Rs1.1Mn

Question#47
A business had following balances for its plant and machinery .
Rs in Mn
Opening 3.5
Additions during the year 0.5
Disposals during the year ?
Closing 2.9
What is the amount of disposals during the year?
A. Rs4.0Mn B. Rs6.9Mn
C. Rs1.1Mn D. Rs3.5Mn

Question#48
Business owner made a profit during year of Rs950,000 and made drawings of Rs120,000 .
What is the net impact on equity?
A. Rs950,000 B. Rs830,000
C. Rs120,000 D. Rs1,070,000

Question#49
This is the account of equity of Jamil Textiles as at 30 June 2021.
Rs in Mn
Opening 3.1
Profit for the year 1.2
Drawings 0.6
Closing ?
Calculate the amount of closing equity?
A. Rs4.3Mn B. Rs4.9Mn
C. Rs3.7Mn D. Rs2.5Mn

Question#50
This is the account of equity of Jamal Textiles as at 30 June 2021.
Rs in Mn
Opening 5.9
Profit for the year 4.5
Drawings ?
Closing 8.3
Work out the amount to be posted in drawings?
A. Rs2.1Mn B. Rs9.7Mn
C. Rs18.7Mn D. Rs4.0Mn

Prepared by Maaz Imtiaz


8
Prepared by Maaz Imtiaz
9
Chapter 2 Books of prime entry
Question#1
Records sales on credit (receivables) from sales invoices.
A. Journal B. Cash book
C. Credit note D. Sales day book

Question#2
Records transactions that are not recorded in any of the other books of original entry.
A. Statement of financial position B. Journal
C. Bank book D. Cash book

Question#3
An account which summarizes a large number of transactions.
A. Journal ledger B. Trial balance
C. Control Account D. T-Account

Question#4
The receivables control account and the receivables ledger are ?
A. Not updated at same time B. Updated at the same time
C. Updated when required D. Updated at year end

Question#5
The process of transferring the details of transactions from the books of prime entry to the
accounts in the ledgers is called _________ the transactions.
A. Recording B. Updating
C. Posting D. Booking

Question#6
Electric store has made following sales :
Credit sales: Rs1000
Cash sales : Rs500
Which recording of transaction is correct?
A. Cash receipt book Rs.1500 B. Sales day book Rs.1500
C. Account receivable Rs.1500 D. Sales day book Rs.1000 , Cash receipt
book Rs.500

Question#7
Return inwards are recorded in :
A. Cash book B. Goods dispatch note
C. Sales return D. Purchase return

Question#8
Which is not book of prime entry?
A. Trial balance B. Sales day book
C. Purchase day book D. Sales return day book

Question#9
Credit note is issued when?

Prepared by Maaz Imtiaz


10
A. Supplier issues goods B. When customer return goods
C. Supplier dispatches goods D. When customer accepts goods

Question#10
A debit note is issued when ?
A. Supplier issues goods B. When customer return goods
C. Supplier dispatches goods D. When supplier return goods

Question#11
Receipt from a debtor worth Rs.800,000 be recorded in ?
A. Accounts Receivable B. Petty cash book
C. Cash book D. None

Question#12
Payment to a supplier worth Rs.800,000 be recorded in ?
A. Accounts Receivable B. Cash payment journal
C. Cash book D. Petty cash book

Question#13
Which one is current liability?
A. Accounts receivable B. Accrued salaries
C. Rent prepaid D. Inventory

Question#14
Mr. Mushtaq , being a owner, took goods for his own use , how this will be recorded ?
A. Stock Dr , Drawings Cr B. Drawings Dr , Purchases Cr
C. Drawings Dr , Sales Cr D. Purchases Dr , Drawings Cr

Question#15
Maaz sold goods to Shahab on credit. What entries should Maaz make in his ledger ?
A. Account Receivables Dr , Sales Cr B. Cash Dr , Sales Cr
C. Cash Dr , Income Cr D. Account Receivable Dr , Cash Cr

Question#16
A businessman maintains petty cash book . The amount is Rs. 500. During a month, payments
totaling Rs. 300 were made. How much amount will be reimbursed at the end of the month to
restore the petty cash?
A. Rs 300 B. Rs 200
C. Rs 500 D. Rs 800

Question#17
Which of the following document is prepared by the store keeper on receiving goods in store?
A. Goods Dispatch Note B. Goods Delievery Note
C. Debit Note D. Goods Recieve Note

Question#18
Mr Taha , a trader prepares a ledger account using the running balance method .
Which statement is correct ?
A. Different from traditional T-account B. After each transaction the balance is
shown

Prepared by Maaz Imtiaz


11
C. This method is not suitable for trader D. Balance is shown only at month end

Question#19
What is the accounting equation ?
A. Assets + Liabilities = Capital B. Capital + Assets = liabilities
C. Assets – Liabilities = Capital D. Capital-Assets = Liabilities

Question#20
Maaz return goods to Saad which he bought on credit . How is this recorded in Maaz’s books ?
A. Accounts Receivable Dr , Sales Return Cr B. Sales Return Dr , Accounts Receivable Cr
C. Purchase returns Dr , Accounts D. Accounts Receivable Dr , Purchase
Receivable Cr returns Cr

Question#21
A business man provided some information .
Sales Rs 120,000
Cost of Sales Rs 48,000
Expenses Rs 43,000
What is net profit ?
A. Rs 5,000 B. Rs 72,000
C. Rs 29,000 D. Rs 63,000

Question#22
How is capital employed calculated?
A. Fixed assets + current assets + current B. Fixed assets – current assets + current
liabilities liabilities
C. Fixed assets + current assets – current D. Fixed assets – current assets – current
liabilities liabilities

Question#23
A transport company received Rs500,000 from passengers . The sold a van for Rs5,000,000 . How
these receipts are treated?
A. Capital receipts: Rs5,000,000 ; Revenue B. Capital receipts: Rs500,000 ; Revenue
receipts : Rs500,000 receipts : Rs5,000,000
C. Capital receipts: Rs--- ; Revenue receipts D. Capital receipts: Rs5,500,000 ; Revenue
: Rs5,500,000 receipts : Rs---

Question#24
Which does not reduce the owner’s capital?
A. Personal expenses paid from business B. Cash withdrawn from the business bank
account account for personal use
C. Goods taken from inventory for D. Business expenses paid using a personal
personal use cheque

Question#25
A eatery owner spends the following amounts :
Rs
Plates 1,500,000
Food 5,000,000
Cutlery 500,000

Prepared by Maaz Imtiaz


12
Drinks 3,000,000
What is the amount of capital expenditure?
A. Rs10,000,000 B. Rs7,000,000
C. Rs2,000,000 D. Rs8,000,000

Question#26
Businesses use book-keeping and accounting procedures .
What is the purpose of book-keeping?
A. To analyse accounting records B. To record all financial and other
transactions
C. To prepare financial statements D. To record all financial transactions

Question#27
Which would be recorded by a debit entry in an account?
A. Increase in capital B. Decrease in assets
C. Decrease in liabilities D. Increase in liabilities

Question#28
Maaz decides to write off a debt due to him from Saad for goods sold on credit. How is this
recorded in Maaz’s books?
A. Bad debts : Dr , Saad(recievables) : Cr B. Sales : Dr , Bad debts : Cr
C. Bad debts : Dr , Sales : Cr D. Saad(recievables) : Dr , Bad debts : Cr

Question#29
Asma provided the following information at end of her financial year .
Rs
Non current assets at cost 14,000,000
Amount owning by customers 1,250,000
Amount owning to suppliers 850,000
Inventory 1,875,000
Bank overdraft 365,000
What was Asma’s capital?
A. 15,910,000 B. 16,640,000
C. 12,160,000 D. 15,110,000

Question#30
Maaz enterprises provided some information from its records .
Rs
Trade recievables 13400
Provision for doubtful debts b/f 800
Written off bad debts 600
Provision for doubtful debts to be ?
carried forward @ 5%
What is the amount to be included in provision for doubtful debts to be carried forward to next
year?
A. 640 B. 650
C. 700 D. 770

Question#31
Shahab Industries deals in manufacturing and supply of chocolate conching machine and

Prepared by Maaz Imtiaz


13
maintains provision for doubtful debts @2.5% . On 1 July the balance on the provision account
was Rs1075. The trade receivables at 30 June amounted to Rs41,000 .
Which entry will accountant of Shahab Industries will make on 30 June to adjust provision for
doubtful debts?
A. Provision for doubtful debts : Rs1025 Dr B. Provision for doubtful debts : Rs50Dr ,
, income statement : Rs1025Cr income statement : Rs50Cr
C. income statement : Rs1025Dr , D. income statement : Rs50Dr , Provision
provision for doubtful debts : Rs1025 Cr for doubtful debts : Rs50Cr

Question#32
Omar had the following assets and liablities at 1 September 2021 .
Rs
Non current assets 44,000
Non current liablities 10,000
inventory 1,500
Trade payables 3,300
Trade recievables 2,600
What was Omar’s capital?
A. 35,800 B. 44,000
C. 34,800 D. 54,800

Question#33
Kamran made following transactions during a certain month: Credit sales 30,000 out of which Rs.
6,000 were returned by the customers subsequently Cash sales amounting Rs. 16,000 Cash
received from customers Rs. 8,000 What is the total of sales day book for the month?
A. 40,000 B. 46,000
C. 30,000 D. 16,000

Question#34
Naeem runs a business of purchase and sales of furniture. During a particular period Naeem made
following transactions:
Sales to A Rs. 24,000
Sales to B Rs. 20,000 list price and trade discount 3%
Sales returns from A Rs. 6,000. What is the total of sales day book for the period?
A. 38,000 B. 43,400
C. 37,400 D. 44,000

Question#35
A confectionary shop made following transactions during a certain period:
Credit purchases Rs. 28,000; out of which Rs. 5,000 were rejected and subsequently returned to
the supplier. Supplier allowed a discount of 2% on settlement of amount. What is the net amount
of cash paid to the supplier?
A. 23000 B. 22,310
C. 22,540 D. 20,220

Question#36
An Account receivable control ledger account had a closing balance of Rs. 900,000. This control
account receivable ledger contained a contra to the account payable ledger of Rs. 110,000; but it
had been posted to wrong side of control account.
What is the correct closing balance in account receivable control ledger account after correction of

Prepared by Maaz Imtiaz


14
the above?
A. 790,000 B. 900,000
C. 680,000 D. 1,010,000

Question#37
A receivable control account contains the following relevant entries for the year ended 31
December 2021:
Rs.
Receiving through bank 170,000
Credit sales during the year 190,000
Discount received 18,000
Contra with payable control account 5,000
Balance c/d as on Dec 31, 2021 25,000
There are no other transactions related to account receivable.
What is the amount of balance b/d of account receivable control account as on 1 January 2021?
A. 408,000 B. 28,000
C. 398,000 D. 358,000

Question#38
Maaz bought goods on credit from Noman Traders (NT). The goods were not according to the
required specifications and therefore returned to NT. Which document should Maaz send to NT?
A. Invoice B. Credit note
C. Receiving acknowledgement slip D. Debit note

Question#39
In which book of prime entry does credit note received worth Rs.800,000 would be recorded?
A. Sales day book B. Return outwards journal
C. Purchase return day book D. Return inwards journal

Question#40
In which book of prime entry does receipt from a debtor worth Rs.600,000 enter?
A. Cash book B. Petty cash book
C. Purchase day book D. Sales day book

Question#41
A business man provided following information
Rs
Cost of sales 80,000
Expenses 7,000
Profit for the year 15,000
What is the revenue?
A.102,000 B.87,000
C.22,000 D.95,000

Question#42
A eatery owner spends the following amounts :
Rs
Plates 1,500,000
Food 5,000,000
Cutlery 500,000

Prepared by Maaz Imtiaz


15
Drinks 3,000,000
What is the amount of capital expenditure?
A. Rs10,000,000 B. Rs7,000,000
C. Rs2,000,000 D. Rs8,000,000

Question#43
Which entry is made when provision for doubtful debts is decreased ?
A. Trade recievables Dr , Provision for B. Provision for doubtful debts Dr , Trade
doubtful debts Cr recievables Cr
C. Provision for doubtful debts Dr , Income D. Income statement Dr , Provision for
statement Cr doubtful debts Cr

Question#44
China toys , a wholeseller had following transactions :
Sold goods worth Rs30,000 on credit .
Received cheque of Rs12,000 from sale of old plant at book value .
How these transactions should be classified ?
A. Revenue receipt 42,000 , Capital receipt B. Revenue receipt 30,000 , Capital receipt
--- 12,000
C. Revenue receipt 12,000 , Capital D. Revenue receipt --- , Capital receipt
receipt 30,000 42,000

Question#45
Younus maintains provision for doubtful debts of 3% of the trade receivables at the end of year .
On 30 June 2020 trade recievables amounted to Rs 70,000 .
On 30 June 2021 trade recievables amounted to Rs 73,000 .
Which double entry should be made by Younus at 30 June 2021 ?

A. Debit : Income statement 2,090 , Credit B. Debit : Provision for doubtful debts
: Provision for doubtful debts 2,090 2,090 , Credit : Income statement 2,090
C. Debit : Income statement 90 , Credit : D. Debit : Provision for doubtful debts 90 ,
Provision for doubtful debts 90 Credit : Income statement 90

Question#46
A motor vehicle having book value of Rs 2700 is sold for Rs 3150 .
How is this transaction is treated ?
A. Capital receipt :2700 , B. Capital receipt :3150 , Revenue receipt :
Revenuereceipt:450 _____
C. Capital receipt :450 , D. Capital receipt :_____ , Revenue receipt
Revenuereceipt:2700 :3150

Question#47
How can purchases be calculated ?
A. Cost of goods sold + closing inventory + B. Cost of goods sold + closing inventory –
opening inventory opening inventory
C. Cost of goods sold – closing inventory + D. Cost of goods sold – closing inventory –
opening inventory opening inventory

Question#48
Which is liablity of a business ?

Prepared by Maaz Imtiaz


16
A. Property tax paid in advance B. Long term loan to employee
C. Amount owning to credit suppliers D. Amount owning by credit customers

Question#49
Following information relates to Sales transaction of Nadir during November 2021
 Receivables as at 1 November Rs. 75,000
 Receivables as at 30 November Rs. 47,000
 Receipts during the period (after allowing discount of Rs. 8,000) Rs. 55,000
What is the amount of credit sales during November?
A. 27,000 B. 35,000
C. 102,000 D. 27,800

Question#50
Which of the following items will appear on the debit side of a payable control ledger?
A. Balance b/d B. Refunds from a credit supplier
C. Cash purchases D. Payment to credit supplier

Prepared by Maaz Imtiaz


17
Chapter 3 Ledgers and Trial balance
Question#1
Running account account balance provides :
A. Updated balance after each transaction . B. Balance at period end
C. Balance at month end D. Updated balance at month end

Question#2
The process of classifying amounts from books of prime entry to ledgers is called:
A. Recording B. Posting
C. Booking D. All of above

Question#3
What is the effect on final accounts of providing for a salaries accrual?
A. Net profit : Increase , Current liablities : B. Net profit : Decrease, Current liablities :
Increase Increase
C. Net profit : Increase , Current liablities : D. Net profit : Decrease, Current liablities :
Decrease Decrease

Question#4
What is the purpose for the trial balance?
A. To show transactions B. To confirm that total debit balance agree
with total credit balance
C. To calculate provisional profit D. To ascertain asset and liablities

Question#5
A junior accountant asked senior accountant what is a purpose of trial balance?
A. Summarising transactions B. Provide details for assets and liablities
C. Provide financial information D. It is a useful means of checking for errors
in the accounting system .

Question#6
Which will appear in credit column of trial balance?
A. Cash at bank B. Discounts received
C. Total assets D. Discounts allowed

Question#7
Which will appear in debit column of trial balance?
A. Cash at bank B. Discounts received
C. Accrued rent D. Sales

Question#8
Which error is not shown by trial balance?
A. If a transaction completely omitted from B. Error made while calculating total

Prepared by Maaz Imtiaz


18
accounting records
C. Only single entry for transaction is made D. Error while totaling credit side of trial
balance

Question#9
Why does a trial balance is prepared by business?
A. To calculate amount of total profit B. To calculate amount of total profit , assets
, liablities and equity
C. To calculate amount of total profit , assets D. To check mathematical accuracy of ledger
and liablities

Question#10
By which error totals of trial balance will disagree?
A. Sales made on credit had been credited to B. Fixtures purchased by cheque were not
account of wrong customer recorded
C. Fixtures purchased by cheque were D. Sales made on credit had been debited to
recorded as an asset account of wrong customer

Question#11
Mr Aslam takes goods from business for personal use.
How this will effect profit for the year and current assets?
A. Profit for the year : decrease , Current B. Profit for the year : decrease , Current
assets : No effect assets : decrease
C. Profit for the year : no effect , Current D. Profit for the year : no effect , Current
assets : No effect assets : decrease

Question#12
Which is not the limitation of trial balance ?
A. Errors of omission in the books of original B. Errors of principle (posting on right side of
record wrong account nature wise)
D. Checking for errors in the accounting
C. Incorrect account in the original books (on system.
both sides )

Question#13
Process by which transactions are transferred from journal to ledgers is called?
A. Summarising B. Analysing
C. Recording D. Posting

Question#14
Balance on debit side is 18,000 and balance on credit side is 28,000 . What is the closing balance and
on which side?
A. 46,000 on right side B. 10,000 on right side
C. 46,000 on left side D. 10,000 on left side

Question#15

Prepared by Maaz Imtiaz


19
Which of the following statement is correct?
A. T-account and running balance account B. T-account and running balance account
are balanced periodically are balanced after each transactions
C. T-account is balanced periodically while D. T-account is balanced after each
running balance account is balanced after transactions while running balance
each transactions account is balanced periodically

Question#16
Which one of the following account would usually have a debit balance?
A. Cash B. Accounts payable
C. Owner equity D. Bank loan

Question#17
The recievables account is part of:
A. Sales ledger B. Recievables ledger
C. General ledger D. Purchase ledger

Question#18
A trial balance which shows equal totals for debit and credit columns?
A. Shows that there must be more than one B. Shows that there is only one error within
error within the bookkeeping system the bookkeeping system
C. Shows that the bookkeeping system is D. Shows that the arithmetic is correct , but
free from errors errors may still be present within the
bookkeeping system

Question#19
A business maintains running balance account ledger. A supplier had balance of Rs. 55,000 credit. A
further purchase of Rs. 30,000 was made from him and a business paid Rs. 32,000 by cheque. What
would be his account balance after posting all transactions?
A. Rs53,000 credit B. Rs57,000 credit
C. Rs53,000 debit D. Rs57,000 debit

Question#20
Which of the following accounts normally have credit balance?
A. Return inwards B. Motor vehicles
C. Prepaid rent D. Wages outstanding

Question#21
Each invoice value from purchase journal is posted to:
A. Credit side of individual supplier account B. Debit side of individual supplier account
in payables ledger in payables ledger
C. Credit side of individual supplier account D. Debit side of individual supplier account
in purchase ledger in general ledger

Question#22

Prepared by Maaz Imtiaz


20
The Return outwards account would be found in
A. Purchase ledger B. Nominal ledger
C. Purchase return ledger D. Sales ledger

Question#23
Talha gives a cash discount of Rs80 toa customer . The discount is credited to the discounts allowed
account . The effect of recording the discount in this way is that profit will be?
A. Understated by 80 B. Understated by 160
C. Overstated by 160 D. None of above

Question#24
The inventory account is part of:
A. Purchase ledger B. Sales ledger
C. Asset ledger D. General ledger

Question#25
Which of the following best describes trial balance?
A. It is a special account B. Shows financial position of business
C. It is the list of total /balances of books D. Shows all entries in the books

Question#26
Balance on debit side is 90,000 and balance on credit side is 55,000 . What is the closing balance on an
account and on which side?
A. 145,000 on right side of account B. 35,000 on left side of account
C. 35,000 on right side of account D. 145,000 on left side of account

Question#27
Balance on debit side is 83,000 and balance on credit side is 105,000 . What is the closing balance on
an account and on which side?
A. 188,000 right side of account B. 188,000 left side of account
C. 22,000 on right side of account D. 22,000 on left side of account

Question#28
Each return outward (debit note issued) from sales return journal is posted to:`
A. Debit side of individual supplier account B. Debit side of individual supplier account
in Recievables Ledger. in Purchase Ledger.
C. Debit side of individual supplier account D. Debit side of individual supplier account
in Payables Ledger. in Payables Ledger.

Question#29
Which one of the following would not be classified as revenue?
A. Income from sale of fixed assets B. Fee from sale of services
C. Sales on credit D. Sales on cash

Question#30

Prepared by Maaz Imtiaz


21
Suspense account in the trial balance is entered in the?
A. Income statement B. Balance sheet
C. Profit and loss A/c D. TradingA/c

Question#31
Wages paid to workers for the installation of new machine should be debited to?
A. Fixed overheads account B. Factory expenses account
C. Machinery account D. Wages account

Question#32
At the end of accounting period, a company has a profit of Rs500,000 , it shall be transferred to:
A. Debit side of asset account B. Debit side of capital account
C. Credit side of capital account D. None

Question#33
Which of the following statements regarding trial balance is correct?
A. Capital are represented by debit balance B. Every credit balance represents liablity
C. Every debit balance represents an asset . D. Expenses are represented by debit
balances

Question#34
Chart of accounts has following characterstics , identify correct statement .
A. It ensures that all transactions are B. Each account is identified by unique code
recorded according to business and heading
requirement
C. Both A and B D. None

Question#35
A receivable ledger account at 1 September 2020 had balances of Rs. 46,652 debit.
During the year, sales amounting to Rs. 245,000 were made on credit.
Receipts from customers amounted to Rs. 165,900 and cash discounts of Rs. 1,200 were allowed.
What should be the closing balance at 30 September 2020 of account receivable account?
A. 124,892 B. Rs124,552
C. 125,682 D. Rs125,752

Question#36
A payable account of Yosuf Traders contains the following relevant entries for the year ended 30 June
2020:
Rs.
Payment through bank 195,000
Credit purchases during the month 205,600
Discount received 28,900

Prepared by Maaz Imtiaz


22
Contra with receivable account 12,700
Balance c/d as on June 30, 2020 35,450
What is the amount of balance b/d of account payable account as on 1 June 2020?
A. Rs66,450 B. Rs41,050
C. Rs53,750 D. Rs55,250

Question#37
At the end of accounting period, a company has a net loss of Rs650,000 , it shall be transferred to:
A. Credit side of asset account B. Debit side of capital account
C. Credit side of capital account D. None

Question#38
Zara’s account payable ledger has the following balances as at 1 June 2020 :
Rs
Opening balance 138,000
Credit purchases 55,600
Cash purchases 13,900
Cheque issued to credit suppliers ( after discount 100,600
of Rs5,500)
What is the amount of closing balance at 30 June 2020?
A. Rs18,200 B. Rs87,500
C. Rs193,600 D. RS92,900

Question#39
A company had a cash balance of Rs. 250,000 at the start of the month. During the month, the
following transactions occurred.
Rs
Payments to trade payables 58,000
Purchases on credit 98,600
Sales on credit 68,000
Cash from trade recievables 92,000
A. Rs352,000 B. Rs400,000
C. Rs253,400 D. Rs284,000

Question#40
Nasir has extracted the following balances from his general ledger at 30 June 2020:
Rs.
Sales 357,156

Prepared by Maaz Imtiaz


23
Purchases 159,763
Expenses 46,325
Non-current assets (carrying amount) 69,578
Receivables 29,453
Payables 17,548
Cash at bank 2,548
Capital 13,155
What is the total of the debit balances in Nasir's trial balance at 30 June 2020?
A. Rs215,017 B. Rs261,342
C. Rs201,862 D. Rs307,667

Question#41
A recievable account of Younus Traders contains the following relevant entries for the year ended 30
June 2020:
Rs.
Payment in bank 563,890
Credit sales during the month 452,880
Discount allowed 28,900
Contra with payable account 67,580
Balance c/d as on June 30, 2020 350,950
What is the amount of balance b/d of account payable account as on 1 June 2020?
A. Rs490,860 B. Rs558,440
C. Rs423,280 D. RS500,640

Question#42
A total of trial balance has Rs600,500. A machinery was purchased for Rs55,000 on credit but remain
unrecorded. How this will impact both sides of trial balance when machinery is recorded.
A. Rs55,000Dr , Rs55,000 Cr B. Rs55,000 Cr only
C. Rs55,000Dr only D. None

Question#43
A business buys building for Rs40Mn and sells another building for Rs19Mn . At start of year opening
balance on building account was Rs90Mn. What is the amount at year end?
A. Rs130Mn B. Rs111Mn
C. Rs115Mn D. Rs149Mn

Question#44
What is the purpose for the trial balance?
A. To confirm that total debit balance agree B. To ascertain assets and liablities

Prepared by Maaz Imtiaz


24
with total credit balance
C. To calculate provisional profit D. To ascertain assets and liablities and
equity

Question#45
A debit balance is expected to arise when the accounts are balanced at the period end on which of the
following accounts?
A. Rent B. Capital
C. Sales D. Bank loan

Question#46
Running account account balance provides :
A. Upadated balance at period end B. Updated balance at month end
C. Updated balance at year end D. Updated balance after each transactions

Question#47
Which will appear in debit column of trial balance?
A. Prepaid rent B. Accrured rent
C. Trade payables D. Advance rent recieved

Question#48
An entity recorded the following journal entry in its General Journal:
Vehicle Rs. 1,500,000
Maaz Motors (supplier) Rs. 1,500,000
How this would be posted in the ledger account of “Maaz Motors”?
A. On debit side with description “Maaz B. On credit
Motors” and amount Rs. 1,500,000 C. side with description “Maaz Motors” and
amount Rs. 1,500,000
D. On credit side with description “Vehicle” E. On debit side with description “Vehicle”
and amount Rs. 1,500,000 and amount Rs. 1,500,000

Question#49
A credit balance is expected to arise when the accounts are balanced at the period end on which of the
following accounts?

A. Capital B. Prepaid rent


C. Machinery D. Recievables

Question#50
Which error is not shown by trial balance?
A. Error made while totaling of credit side B. Error while totaling credit side of trial
balance
C. Only single entry is made for treansaction D. If a transaction completely omitted from
accounting records

Prepared by Maaz Imtiaz


25
Prepared by Maaz Imtiaz
26
Chapter 4 Accruals and Prepayments
Question#1
Financial statements are prepared on _______ basis of accounting.
A. Accrual B. Cash
C. Prepayment D. None

Question#2
Mr. Aslam paid Rs 50,000 in advance to Mr Atif for cleaning services.
How this income is classified in Mr Atif books .
A. Accrued income B. Unearned income
C. Prepaid income D. B and C

Question#3
Accrued expense means :
A. Prepaid expense B. Paid expense
C. Unpaid expense D. None

Question#4
Mr Zamir has electricity expense of Rs90,000 which he has not paid . How this expense is classified in
Mr Zamir’s books?
A. Accrued expense B. Advance expense
C. Expense recievable D. Prepaid expense

Question#5
Which of the following statement is correct?
1. Accrued expense in recorded in income statement of business
2. Accrued expense is not recorded as liablity in balance sheet of business .
A. 1 is correct B. 2 is correct
C. 1 and 2 are correct D. none

Question#6
Mr Zahir has paid to landlord rent of six months in advance amounting to Rs150,000.
How is this classified?
A. Prepaid rent B. Accrued rent
C. Accrued expense D. Unearned rent

Question#7
A business provided following information related to its accrued rent account :
Rs
Opening 120,000
Rent for the year 500,000
Amount paid during the year 550,000

Prepared by Maaz Imtiaz


27
Closing ?
A. Rs70,000 B. Rs80,000
C. Rs120,000 D. Rs90,000

Question#8
Mr Daniyal paid Rs25,000 in advance for telephone bill , which entry is correct to record this
transaction .
A. Bill expense 25,000 Dr , Bank 25,000 Cr B. Bill expense 25,000 Dr , Cash 25,000 Cr
C. Prepaid bill 25,000 Dr , Bank 25,000 Cr D. Prepaid bill 25,000 Dr , Bill expense
25,000 Cr

Question#9
On 1 April 2020 , Mr Waseem paid advance rent of Rs120,000 for one year . What is the amount to be
included in income statement and balance sheet at 31 December 2020 .
A. Income statement 120,000 , Balance sheet B. Income statement 30,000 , Balance sheet
00 90,000
C. Income statement 90,000 , Balance sheet D. Income statement 00 , Balance sheet
30,000 120,000

Question#10
Mr Rizwan comes to know that the internet fee he paid for the year for his office includes Rs5,000 in
advance . What is correct year end adjustment to be made for advance rent?
A. Internet Fee 5,000 Dr , Accrued Internet B. Internet Fee 5,000 Dr , Internet Fee
Fee 5,000 Cr Expense 5,000 Cr
C. Internet Fee 5,000 Dr , Advance Internet D. Advance internet fee 5,000 Dr , Internet
Fee 5,000 Cr Fee expense 5,000 Cr

Question#11
On year end a shop owner has outstanding electricity bills of Rs. 27,000. During the year electricity
bills paid are Rs. 220,000. What adjustment will be required to utilities expense account regarding the
outstanding bills?
A. Rs27,000 Cr B. Rs220,000 Cr
C. Rs27,000 Dr D. None

Question#12
While closing accounts at year end Mr Abdullah noticed that he has ignored prepayments of Rs 50,000
and accrued expenses of Rs 40,000 . What will be the impact on profit for the year if he did not
discover the fact .
A. Understated by 10,000 B. Undersated by 40,000
C. Overstated by 10,000 D. Overstated by 40,000

Question#13
On finalizing the draft accounts Abdullah identified that he has Rs. 20,000 rental income receivable.
Rental income received and recorded during the year is Rs. 220,000. What is the correct entry to
record the accrued income?
A. Dr Cash Rs. 220,000 Cr Rental income Rs. B. Dr Cash Rs. 240,000 Cr Rental income Rs.

Prepared by Maaz Imtiaz


28
220,000 240,000
C. Dr Accrued income Rs. 20,000 Cr Rental D. Dr Rental income Rs. 20,000 Cr Accrued
income Rs. 20,000 income Rs. 20,000

Question#14
The expense recognition principle matches:
A. creditors with businesses. B. assets with liabilities.
C. expenses with revenues. D. customers with businesses.

Question#15
Which of the following is asset account?
A. Rent payable B. Unearned income
C. Prepaid expense D. Rent payable

Question#16
Which of the following statements is not true?
A. Accruals decrease profit B. A prepayment is an asset
C. Accrued income decreases profit D. All are true

Question#17
Which of the following is liablity account ?
A. Rent payable B. Prepaid expense
C. Rent recieveable D. None

Question#18
On July 1 the Fisher Shoe Store paid Rs24,000 to Acme Realty for 6 months rent beginning July 1.
Prepaid Rent was debited for the full amount. If financial statements are prepared on July 31, the
adjusting entry to be made by the Fisher Shoe Store is:
A. debit Rent Expense, Rs4,000; credit B. debit Prepaid Rent, Rs4,000; credit Rent
Prepaid Rent, Rs4,000. Expense, Rs4,000.
C. debit Rent Expense, Rs24,000; credit D. debit Rent Expense, Rs24,000; credit
Prepaid Rent, Rs20,000. Prepaid Rent, Rs4,000.

Question#19
A law firm has billed their clients for services performed. They subsequently received payments from
their clients. What entry will the law firm make upon receipt of the payments?
A. Debit Unearned Service Revenue and B. Debit Cash and credit Accounts
credit Service Revenue Receivable
C. Debit Accounts Receivable and credit D. Debit Cash and credit Service Revenue
Service Revenue

Question#20
If a company fails to adjust an Unearned Rent Revenue account for rent that has been recognized,
what effect will this have on that month’s financial statements?
A. Assets will be overstated and revenues B. Liabilities will be overstated and revenues
will be understated. will be understated.

Prepared by Maaz Imtiaz


29
C. Liabilities will be understated and D. Assets will be understated and revenues
revenues will be understated. will be understated.

Question#21
If a business have received cash in advance of services performed and credits a liability account , the
adjusting entry needed after the services are performed will be :
A. Dr unearned service revenue , Cr cash B. Dr unearned service revenue , Cr service
revenue
C. Dr unearned service revenue , Cr Prepaid D. Dr unearned service revenue , Cr
expense Accounts recievable

Question#22
Ms Asma paid Rs30,000 as insurance premium for her factory vehicle at 1 March 2019 . What is the
amount to be charged in Income statement and balance sheet at 31 December 2019.
A. Income statement 25,000 , Balance sheet B. Income statement 25,000 , Balance sheet
25,000 5,000
C. Income statement 30,000 , Balance sheet D. Income statement 5,000 , Balance sheet
30,000 25,000

Question#23
Mr Tahir has accrued rent expense of Rs28,000 on rent account at the start of year . During the year
rent expense was Rs45,000 . At the end of year accrued rent expense was Rs35,000 . What is the
amount of cash he paid to landlord during the year?
A. Rs.35,000 B. Rs.28,000
C. Rs.36,000 D. Rs.38,000

Question#24
Mr Yousuf a tenant pays annual rent of Rs. 120,000. Payment is made quarterly in advance on 1
January, 1 April, 1 July and 1 October. Which of the following should be included in his accounts for
the year ended 31 October 2001?
A. Rs20,000 Prepayment B. Rs20,000 accrual
C. Rs10,000 Prepayment D. Rs10,000 accrual

Question#25
Given the data below for a firm in its first year of operation, determine net income under the accrual
basis of accounting.
Rs
Revenue recognized 19,000
Accounts recievable 3,000
Expenses incurred 7,250
Accounts payable (related to expenses) 750
Supplies purchased with cash 1,800

A. 12,200 B. 19,500
C. 14,000 D. 11,750

Prepared by Maaz Imtiaz


30
Question#26
How unearned income is classified in statement of financial position?
A. Current asset B. Non current asset
C. Current liablities D. Non current liablities

Question#27
A Company had the following transactions during 20X1.
 Sales of Rs900,000 on account
 Collected Rs400,000 for services to be performed in 20X2
 Paid $265,000 cash in salaries
 Purchased airline tickets for Rs50,000 in December for a trip to take place in 20X1
What is the company’s net income using cash basis of accounting.
A. Rs985,000 B. Rs85,000
C. Rs135,000 D. Rs1,035,000

Question#28
In the year to 30 June 2019, Kashif received Rs. 130,500 rental income. The amounts of rent received
in advance and due in arrears were as follows:
30 June 2019 30 June 2018
Rs Rs
Rent received in advance 40,500 35,700
Rent in arrears(accrued) 28,900 19,550
What figure for rental income should be recorded in the statement of profit or loss for the year ended
30 June 2019?
A. Rs130,000 B. Rs135,050
C. Rs130,500` D. Rs155,050

Question#29
What is the treatment of advance income in the Statement of Financial Position of the business?
A. Non current asset B. Current asset
C. Non current liablity D. Current liablity

Question#30
The annual rent expense for a factory building for the period 1 March 2019 to 28 Feburary 2020 is Rs.
690,000, which is 10% more than the previous year. Rent expense is paid on 1 March. What is the
charge of rent expense in the statement of profit or loss for the year ended 31 December 2019?
A. Rs690,000 B. Rs679,546
C. Rs579,546 D. Rs679,446

Question#31
A superstore has sublet part of its warehouse , in the year ended 31 October 2020 the rent receivable
was:
Until 30 June 2020 Rs. 1,200,000 per year
From 1 July 2020 Rs. 1,500,000 per year
Rent was received quarterly in advance on 1 January, April, July, and October each year.
What amount of rent company has recieved for the year ended 31 October 2020?

Prepared by Maaz Imtiaz


31
A. Rs 1,200,000 B. Rs 1,300,000
C. Rs 1,500,000 D. Rs 1,400,000

Question#32
Which of the following statements is true?
A. Accrued income decrease profit B. Accruals increase profit
C. Accrued income increase profit D. Both B and C

Question#33
If an accrual as at the end of Rs1,500 was treated to a prepayment , the net profit for the year would
be?
A. Understated by Rs3,000 B. Overstated by Rs3,000
C. Understated by Rs1,500 D. Overstated by Rs1,500

Question#34
Expenses relevant to the accounting period which remain unpaid by period end should be:
A. Not considered until paid B. Recorded only when paid
C. Recorded as expenses in the income D. Recorded as expenses in the income
statement for the period to which it statement for the period to which it
pertains and recorded as asset in the pertains and recorded as liability in
balance sheet of same period. balance sheet of same period.

Question#35
An accountant has treated prepaid insurance of Rs. 50,000 as accrued expense. What will be the
impact of correction?
A. Profit will be reduced by Rs. 2,000 B. Profit will be reduced by Rs. 1,000
C. Profit will be increased by Rs. 1,000 D. Profit will be increased by Rs. 2,000

Question#36
Which of the following statements is correct?
A. Prepaid insurance is not a current asset B. Accrued income is not a liability
C. Advance income is not a liability D. Prepaid insurance is a non current asset

Question#37
On its Statement of financial position a business would report as a current liability the amounts:
A. It intends to pay immediately after B. It hopes to repay one day.
balance sheet date.
C. It intends to repay within one year of the D. It intends to pay as early as possible but
balance sheet date. not later than a month after the balance
sheet date.

Question#38
A statement of financial position would show prepayments as
A. Current assets B. Non-current assets
C. Current liabilities D. Noncurrent liabilities

Prepared by Maaz Imtiaz


32
Question#39
Staff salary remaining unpaid as at the year-end should be accounted for as:
A. Debit salary accrued account and credit B. Debit Prepaid salary account and credit
staff salary account staff salary account .
C. Debit staff salary account and credit Cash D. Debit staff salary account and credit
account . salary accrued account

Question#40
Which of the following is correct: Where accounting records are maintained on accrual basis:
A. Income should accounted for only when it B. Expenditure should be accounted for only
is received when it is paid
C. Income and expenditure relating to the D. Income should be accounted on accrual
accounting period should be fully basis and expenditure on payment basis .
accounted for even if income is still to be
received and expenditure is yet to be paid
for .

Question#41
The draft year end accounts were prepared without adjusting prepayment for rent of Rs. 5,000. When
the adjustment is made, which of the following would be effect thereof?
A. Profit increased by Rs. 5,000. and assets B. Profit increased by Rs. 5,000. and assets
decreased by Rs. 5,000. increased by Rs. 5,000.
C. Profit decreased by Rs. 5,000. and Liability D. Profit increased by Rs. 5,000. and Liability
increased by Rs. 5,000. increased by Rs. 5,000.

Question#42
A company has taken a loan from a bank. Interest on the loan is payable every quarter on March 31,
June 30, September 30 and December 31. Principal amount of the loan is 1,200,000 and the rate of
interest is 8% per annum. Due to financial problems, two instalments were not paid on September 30
and December 31. What is the amount of the interest liability to be shown in the Statement of
Financial Position prepared as on December 31?
A. Rs96,000 B. Rs24,000
C. Rs48,000 D. Rs64,000

Question#43
Prior to preparing the Statement of financial position as at 31 December 2020 adjustments are needed
for :
 Rent income account include Rs80,000 prepaid .
 Office supplies remains unused amounting to Rs25,000
 As at the year end salary accrued and insurance prepaid amount to Rs22,500 and Rs4,800
respectively
The effect of making these adjustments would be
To include in the Statement of financial position an additional
A. Current asset 29,800 ; Current liablity B. Current asset 29,800 ; Current liablity
102,500 29,800

Prepared by Maaz Imtiaz


33
C. Current asset 102,500 ; Current liablity D. Current asset 102,500 ; Current liablity
102,500 29,800

Question#44
A loan of Rs45,000 at 10% interest per year, was given to a member of staff, in the previous year. No
interest has been received during the year. The accounting entries for accruing interest income are:
A. Debit staff loan account and credit B. Debit cash account and credit interest
interest earnings account earning account
C. Debit interest receivable account and D. Debit interest receivable account and
credit interest earnings account credit staff loan account

Question#45
A retailer paid Rs75,000 as rent and treated the whole amount as expenditure for the year,
overlooking the fact that the amount was for a five year period commencing from the beginning of
that year. The effect of this error would be:
A. Net profit and current assets are B. Net profit and current assets are
understated by Rs 60,000 understated by Rs 75,000
C. Net profit and current assets are D. Net profit and current liablities will be
overstated by Rs 60,000 understated by Rs 75,000

Question#46
A business earns income by renting out properties. During the year ended 31 December 2020 it
received Rs38,400 as rent. This amount includes Rs5,400 received as advance for 2021. During 2021 it
received Rs34.500 as rent and a third of this amount relates to 2016. The financial statements for the
year ended 31 December 2020 should report
A. Rental income Rs44,500 , Rent in arrears B. Rental income Rs49,900 , Rent in arrears
Rs11,500 Rs11,500
C. Rental income Rs49,900 , Rent in arrears D. Rental income Rs38,400 , Rent in arrears :
Rs11,500 None

Question#47
Najma pays rent regularly quarterly in arrears on 31st March, 30th June, 30th September and 31st
December. Annual rent, agreed at Rs240,000, was increased to Rs300,000 from 1st July 2020. The
amount to be expensed in the year to 31st August 2021 and reported as accrued as at that date would
be :
A. Expense Rs 240,000 , Accrued Rs50,000 B. Expense Rs 290,000 , Accrued Rs60,000
C. Expense Rs 200,000 , Accrued Rs60,000 D. Expense Rs 300,000 , Accrued Rs50,000

Question#48
The year-end trial balance as at 31st March 2021 reports a debit balance of Rs9,800 in the Insurance
account. This figure includes Rs6,000 paid on 1st January 2021 as insurance for the year ended 31st
December 2021. Ascertain the amount to be charged in the Income Statement as insurance for the
year ended 31st March 2021.
A. Rs15,800 B. Rs6,000
C. Rs5,300 D. Rs9,800

Question#49

Prepared by Maaz Imtiaz


34
A Transporter's trial balance at year end on 31 December 2020 reports the balance in Motor Vehicle
Maintenance account as Rs216,500. This amount includes Rs27,000 paid on 1 August 2020 for servicing
the fleet of vehicles over three years from that date. The amounts to be expensed in 2020 and
reported as prepaid as at the year-end are:
A. Expense Rs189,500 , Prepayment B. Expense Rs239,750 , Prepayment
Rs27,000 Rs23,250
C. Expense Rs193,250 , Prepayment D. Expense Rs216,500 , Prepayment
Rs23,250 Rs27,000

Question#50
A business has a year ended 31 March and receives an invoice for rent of Rs. 750,000 for the six
months to 30 June. What accrual or prepayment is required for the invoice in the year end accounts?
A. Prepaid rent Rs750,000 B. Accrued rent Rs750,000
C. Prepaid rent Rs375,000 D. Accrued rent Rs375,000

Prepared by Maaz Imtiaz


35
Chapter 5 Bad and doubtful debts
Question#1
An accounts receivable that seems collectible without any difficulty are :
A. Bad debts B. Provision for doubtful debts
C. Bad debt expense D. Good debts

Question#2
An accounts receivable that is not collectible are :
A. Bad debts B. Provision for doubtful debts
C. Bad debt expense D. Good debts

Question#3
The figures show a calculation of the provision for doubtful debts.
1 July 2017 30 June 2018
Rs Rs
Trade receivable X 750,000 nil
Trade receivable Y 1,000,000 2,000,000
Trade receivable Z nil 1,500,000
1,750,000 3,500,000
General provision 4,150,000 7,200,000
Total provision 5,900,000 10,700,000
During the period, X was made bankrupt and a final payment of Rs50,000 was received. What is
the charge forthe year to 30 June 2018 for bad and doubtful debts?

A. Rs5,550,000 B. Rs5,500,000
C. Rs5,750,000 D. Rs5,800,000

Question#4
A company increases its provision for bad debts by Rs16,000 from Rs30,000. What will be the effect of
this adjustment on the year-end balance sheet?
A. Net profit will increase by 16,000 and Net B. Net profit will increase by 16,000 and Net
trade receivables is decreased by 46,000 trade receivables is decreased by 16,000
C. Net profit will decrease by 16,000 and Net D. Net profit will decrease by 16,000 and Net
trade receivables is decreased by 46,000 trade receivables is decreased by 16,000

Question#5
The opening balance of “allowance for doubtful debts account” is Rs12,000 whereas the closing
balance of Receivables account is Rs2,300,000. What amount of allowance for doubtful debts should
be charged to statement of comprehensive income using a 4% allowance for doubtful debts for the
current accounting period?
A. Rs92,000 B. Rs80,000
C. Rs104,000 D. Rs88,000

Question#6

Prepared by Maaz Imtiaz


36
A Company’s year-end Sales Ledger balances are shown below.
Debit 142,400
Credit 9,600
When preparing the annual accounts, it was decided to write off bad debts of Rs2,000 and to maintain
the provision for doubtful debts at 3%. What will be the provision for doubtful debts at the year-end?
A. Rs4,212 B. Rs4,000
C. Rs4,322 D. Rs4,522

Question#7
At the beginning of the year a company has a provision for doubtful debts of Rs1,000. At the end of
the year required provision is Rs2,500. During the year debts of Rs1,500 are written off and Rs100 is
received in respect of a debt written off many years ago. What is the net amount charged to the
Income statement for bad and doubtful debts?
A. Rs2,500 B. Rs2,600
C. Rs2,900 D. Rs3,000

Question#8
At 30 , June , 2020 an entity’s receivable totalled Rs. 700,000 and an allowance for bad and doubtful
debts of Rs. 60,000 had been brought down from last year.
It was decided to write off the debts totalling Rs. 30,000 and to adjust allowance for receivable @ 8%
of the receivable. At what amount receivables are to be shown in statement of financial position as at
30 , June , 2020?
A. Rs616,400 B. Rs617,400
C. Rs620,900 D. Rs556,400

Question#9
A business has closing receivables balance is Rs80,000. It includes one of the accounts receivable
named Zahir, who is going through financial crisis. It is expected that he can pay 60% of his total debt
of Rs10,000. Business has decided to calculate an allowance for doubtful debt at 4%. What is the
amount of allowance to be deducted from receivable in statement of financial position?
A. Rs4,850 B. Rs6,800
C. Rs5,920 D. Rs8,900

Question#10
The writing off of a bad debt is an example of the
A. substance over form concept B. prudence concept
C. matching concept D. going concern concept

Question#11
The table shows information about a business.
Rs
Provision for Doubtful Debts at 1 January 2020 700
Trade receivables at 31 December 2020 (after writing off bad debt of Rs30) 15,000
Charge to Income statement for bad and doubtful debts for the year ended 31 December 200
2020 (including bad debts written off Rs30
What is the percentage provision that has been made for doubtful debts at 31 December 2020?

Prepared by Maaz Imtiaz


37
A. 5.8% B. 6.8%
C. 4.9% D. 3.8%

Question#12
The two methods of accounting for uncollectible accounts are the direct write-off method and the
A. Allowance Method B. Bad Debt Method
C. Net Realizable Method D. Accrual Method

Question#13
Under the direct write-off method of accounting for uncollectible accounts, Bad Debt Expense is
debited
A. when an account is determined to be B. whenever a pre-determined amount of
uncollectible. credit sales have been made.
C. at the end of each accounting period. D. when a credit sale is past due.

Question#14
An aging of a company's accounts receivable indicates that Rs6,000 are estimated to be uncollectible.
If Allowance for Doubtful Accounts has a Rs2,000 debit balance, the adjustment to record bad debts
for the period will require a
A. credit to Allowance for Doubtful Accounts B. debit to Bad Debt Expense for Rs4,000.
for Rs4,000.
C. debit to Allowance for Doubtful Accounts D. debit to Bad Debt Expense for Rs8,000.
for Rs8,000.

Question#15
A debit balance in the Allowance for Doubtful Accounts
A. cannot occur if the percentage of B. indicates that actual bad debt write-offs
receivables method of estimating bad have been less than what was estimated.
debts is used.
C. indicates that actual bad debt write-offs D. is the normal balance for that account.
have exceeded previous provisions for
bad debts.

Question#16
When the allowance method of accounting for uncollectible accounts is used, Bad Debt Expense is
recorded
A. when an account is written off as B. as each credit sale is made.
uncollectible.
C. in the same year as the credit sale. D. in the year after the credit sale is made

Question#17
To record estimated uncollectible accounts using the allowance method, the adjusting entry would be
a
A. debit to Loss on Credit Sales and a credit B. debit to Allowance for Doubtful Accounts
to Accounts Receivable and a credit to Accounts Receivable.
C. debit to Bad Debt Expense and a credit to D. debit to Accounts Receivable and a credit

Prepared by Maaz Imtiaz


38
Allowance for Doubtful Accounts to Allowance for Doubtful Accounts

Question#18
Non-trade receivables should be reported separately from trade receivables. Why is this statement
either true or false?
A. It is false because management can B. It is true because non-trade receivables
decide how to report receivables. do not result from business operations
and should not be included with accounts
receivable.
C. It is false because all current receivables D. It is true because trade receivables are
must be grouped together in one account current assets and non-trade receivables
are long term.

Question#19
Which of the following is the effect on net profit if a business decreases allowance for doubtful debts?
A. It will increase net profit B. It will decrease net profit
C. It will increase net profit and gross profit D. None

Question#20
A firm has not recorded the bad debts by mistake. Which of the following is the effect of bad debts
omission?
A. Net profit would increase B. Net profit would decrease
C. Gross profit would understate D. Gross profit would overstate

Question#21
A trial balance at 30 April 2013, before making end of year adjustments, showed:
Debit (Rs) Credit (Rs)
Trade receivables 17 -
Provision for doubtful debts 800 580
-
At 30 April 2013 it was decided to write off a bad debt of Rs800 and to make a provision for
doubtful debts of 2% of trade receivables. During the year an amount of Rs200 was received from a
customer relating to a debt that was written off in the year ended 30 April 2012.
What was the total bad and doubtful debt expense for the year ended 30 April 2013?

A. Rs1,140 B. Rs940
C. Rs560 D. Rs360

Question#22
The nature of “Allowance for doubtful debt” account is:
A. Liability account B. Expense
C. Asset account D. Contra asset account

Question#23
At the end of accounting period, KL Limited finds out that its total Receivables are Rs. 1,200,000. On

Prepared by Maaz Imtiaz


39
scrutiny of accounts, it turned out that a bad debt amounting to Rs. 22,000 was not recorded in the
books of accounts. Furthermore, having considered the current economic situation, management of
the company decided to increase the allowance for doubtful debts by Rs. 50,000.
Find out what net amount to be expensed out in the statement of comprehensive income?
A. Rs72,000 B. Rs70,000
C. Rs72,050 D. Rs70,050

Question#24
At the beginning of the year a business has a provision for doubtful debts of Rs2 600. At the year end
the provision is to be 5% of trade receivables.
The balance on the Trade receivables Control account at the year-end is Rs69 200, before writing off a
bad debt of Rs480. The business operates a separate Bad Debts accounts.
What is the entry in the Income statement for the provision for doubtful debts?
A. Rs860 credit B. Rs836 credit
C. Rs860 debit D. Rs836 debit

Question#25
Which of the following can be most relevant to calculation of allowance for doubtful debts?
A. Total current liabilities B. Total current assets
C. Total credit purchases D. Total credit sales

Question#26
At July 1, 2020 the balance in allowance for receivable showed Rs. 160,000. At the end of the year it is
decided to write off Rs. 90,000 and adjust allowance for receivable to Rs. 180,000. What will be the
effect of this decision on profit for the year?
A. Increase by Rs. 180,000 B. Decrease by Rs. 270,000
C. Decrease by Rs. 90,000 D. Decrease by Rs. 110,000

Question#27
Which of the following Receivables have highest probability to default on trade debts?
A. 30 to 60 days old Receivables B. 60 to 90 days old Receivables
C. Over 100 days old Receivables D. Current month Receivables

Question#28
A business has received an amount of Rs5,000 from a receivable that had been previously written off
as irrecoverable. What is the correct accounting entry to record the transaction?
A. Dr Cash 5,000 Cr Receivables Rs. 5,000 B. Dr Cash 5,000 Cr Bad and doubtful debts
expense a/c Rs. 5,000
C. Dr Statement of comprehensive income D. Dr Statement of comprehensive income
Cr Cash Rs. 5,000 Rs. 5,000 Cr Receivables Rs. 5,000

Question#29
At the end of a financial period, a business has the following balances.
Rs
Total trade receivables balances 10,620
Bad debt not yet written off 260

Prepared by Maaz Imtiaz


40
Provision for doubtful debts brought forward 460
What should the business do if it wishes to maintain the bad debt provision at 5% of trade
receivables?
A. increase the existing provision by Rs71 B. decrease the existing provision by Rs71
C. increase the existing provision by Rs58 D. decrease the existing provision by Rs58

Question#30
Hamza Enterprises has provided following information;
Rs
Opening receivables 45,000
Credit sales 55,000
Cash sales 10,000
Cash received from customers 35,000
Bad debts written off 2,000
Discount received (separate ledger) 3,000
The business maintains allowance at 2% of receivables each year.
What accounting entry is to be passed to record the increase/ decrease of allowance to the statement
of comprehensive income?
A. Dr Receivables Rs. 360 Cr Bad and B. Dr Bad and doubtful debts expense Rs.
doubtful debts expense Rs. 360 360 Cr Receivables. 360
C. Dr Allowance for doubtful debts Rs. 360 D. Dr Bad and doubtful debts expense Rs.
Cr Bad and doubtful debts expense Rs. 360 Cr Allowance for doubtful debts Rs.
360 360

Question#31
A company has the following balances.
Rs
Trade receivables at 31 December 2013 125,400
Provision for doubtful debts at 1 January 2013 1,800
During the year ended 31 December 2013 bad debts of Rs20 500 were written off. The company
provides for 5% of trade receivables at each year-end.
What is the doubtful debts expense for the year ended 31 December 2013?
A. Rs4 470 B. Rs3 445
C. Rs6 270 D. Rs5 245

Question#32
At year end, the receivable balance on 30 June 2020 is Rs. 93,000. This includes a debt of Rs. 1,800
which needs to be written off.
The business maintains allowance for doubtful debts at 5% of Receivable balance. And this year
allowance has increased by 20% as compared to last year.
What was the balance of Allowance for doubtful debts at 1 July 2019?
A. Rs. 5,476 B. Rs. 3,870
C. Rs. 4,550 D. Rs. 3,800

Question#33
What is the nature of allowance for doubtful debt account?

Prepared by Maaz Imtiaz


41
A. Contra asset account B. An equity
C. An asset D. A liability

Question#34
A business makes a provision for doubtful debts equal to 5% of its trade receivables.
At 31 March 2013 net trade receivables were shown in the Balance Sheet as Rs17 100.
At 31 March 2014 the balance on its Sales Ledger Control account was Rs19 000. In the year ended 31
March 2014 a bad debt of Rs800 had been written off.
How much should be debited in the Income statement for the year ended 31 March 2014 for the
provision for doubtful debts?
A. Rs95 B. Rs55
C. Rs50 D. Rs10

Question#35
On 30 September 2015 a manufacturer’s current assets totalled Rs28 000. The next day only two
transactions took place.
 Inventory bought for cash. The list price of Rs2 000 was subject to a trade discount of 20% and
a cash discount of 5%. Payment was made immediately.
 A bad debt of Rs400 was written off.
What was the total of current assets on 2 October 2015?
A. Rs29 600 B. Rs29 520
C. Rs28 080 D. Rs27 680

Question#36
Is there a difference in bad and double debts?
A. They are synonymous B. Yes, doubtful debt refers to an account
receivable that has been clearly identified
as not being collectible. Whereas a bad
debt is an account receivable that might
become a bad debt at some point in the
future
C. Yes, bad debt refers to an account D. No, they are inter-changeable
receivable that has been clearly identified
as not being collectible. Whereas a
doubtful debt is an account receivable
that might become a bad debt at some
point in the future

Question#37
Bilal has following information available for his business for the year ended 31st December 2017:
 Opening allowance for doubtful debts Rs. 6,000
 Bad debts written off during the year Rs. 4,000
 Bad debts recovered Rs. 1,700
 Closing receivables Rs. 95,000
Closing allowance for doubtful debts 6%.
What is the net charge for bad and doubtful debts for statement of comprehensive income?

Prepared by Maaz Imtiaz


42
A. Rs1,000 B. Rs2,000
C. Rs1,500 D. Rs2,500

Question#38
At January 1, 20189 the balance in allowance for receivable showed Rs. 42,000. At the end of the year
it is decided to write off Rs. 22,000 and adjust allowance for receivable to Rs. 35,000. What will be the
charge for bad and doubtful debts for the year?
A. Rs22,000 B. Rs13,000
C. Rs15,000 D. Rs20,000

Question#39
On January 1, Rs. 1,200,000 of goods are sold with credit terms of 1/10, n/30. How much should the
seller expect to receive if the buyer pays on January 8?
A. Rs1,188,000 B. Rs1,111,000
C. Rs1,118,000 D. Rs1,188,800

Question#40
What is the double entry for recording write-off of any of debts?
A. Bad debt expense (debit) and accounts B. Bad debt expense (debit) and allowance
receivable (credit) for doubtful debts (credit)
C. Allowance for doubtful debts (debit) and D. None of them
accounts receivable (credit)

Question#41
Sorting an entity's accounts receivable into classifications such as current, 1-30 days past due, and 31-
60 days past due etc. is known as the?
A. Ratio analysis B. Trend analysis
C. Aging analysis D. Turnover ratio

Question#42
An irrecoverable debt written off two years ago is unexpectedly recovered and entered in the cash
book. What adjustment, if any, will be necessary?
A. No adjustment B. Credit Suspense account
C. Credit Bad debts expense D. Credit Receivables

Question#43
On September 1, Rs. 900,000 of goods are sold with credit terms of 1/10, n/30. On September 4, the
customer returned Rs. 120,000 of the goods. How much should the seller expect to receive if the buyer
pays on September 9?
A. Rs777,220 B. Rs772,200
C. Rs772,000 D. Rs722,200

Question#44
Saima creates allowance for doubtful debts according to the length of time the debt has been
outstanding. At 31 May 2020 the analysis of accounts receivable balances and the associated
allowance was:

Prepared by Maaz Imtiaz


43
Time debt has been Allowance required Balance at 31 May 2020
outstanding
Less than 31 days Nil 54,200
31 – 60 days 5% of balances 32,500
Over 60 days 49% of balances 12,080
If the balance at 1st June 2019 was Rs. 7,250 what adjustment should be made to the doubtful debts
allowance?
A. An increase of 7,544.2 B. A decrease of 294.2
C. An increase of 294.2 D. A decrease of 7,554.2

Question#45
An increase in the allowance for doubtful debts results in:
A. An increase in net profit B. A decrease in current liabilities
C. A decrease in current assets D. An increase in current assets

Question#46
The turnover in an entity was Rs. 2 million and its accounts receivable were 5% of turnover. The entity
wishes to have an allowance for doubtful debts of 4% of receivables, which would make the allowance
one-third higher than the current allowance. How will the profit for the period affected by the change
in allowance?
A. Profit will be increased by Rs. 1,333 B. Profit will be reduced by Rs. 1,333
C. Profit will be increased by Rs. 1,500 D. Profit will be reduced by Rs. 1,500

Question#47
Which of the following statements concerning an allowance for receivables is INCORRECT?
A. The allowance is usually expected to B. Setting up an allowance for doubtful
increase as the value of sales revenue debts account ensures that receivables
recognized increases as a firm expands are not overstated
C. The receivables account balance is written D. All businesses may create an allowance
off when a specific allowance for doubtful for doubtful debts in case credit sale
debts for that customer is created customers do not pay their debts

Question#48
At 30 June 2019, MPL Traders had an allowance for doubtful debts of Rs. 30,000.
During the year ended 30 June 2020 the MPL Traders wrote off debts totalling Rs. 26,000 and at the
end of the year it is decided that the allowance for doubtful debts should be Rs. 22,000.
What should be included in the Statement of Profit or Loss for bad and doubtful debts?
A. Rs18,000 Debit B. Rs18,000 Credit
C. Rs30,000 Debit D. Rs30,000Credit

Question#49
The allowance for receivables in the accounts at 31 October 2001 was Rs. 9,000. During the year ended
31 October 2002, bad debts of Rs. 5,000 were written off.
The receivables balance at 31 October 2002 was Rs. 120,000 and, based on past experience, the entity
wishes to set the allowance at 5% of receivables.
What is the total charge for bad debts and the allowance for receivables in the statement of

Prepared by Maaz Imtiaz


44
comprehensive income for the year ended 31 October 2001?
A. Rs. 8,000 B. Rs. 5,000
C. Rs. 3,000 D. Rs. 2,000

Question#50
A transfer to close the bad and doubtful debts account is to:
A. The allowance for doubtful debts account B. The trading account
C. The Statement of Profit or Loss D. The statement of financial position

Prepared by Maaz Imtiaz


45
Chapter 6 Depreciation
Question#1
Straight line method recognizes depreciation :
A. usage basis B. Evenly throughout useful life
C. amount of depreciation, calculated as a D. As amount of depreciation reduces as the
fraction of remaining years and sum of life of the asset progresses
years’ digits, reduces as the life of the
asset progresses

Question#2
Assets that have a long useful life and are expected to provide future economic benefits for the entity
over a period of several years are :
A. Assets B. Non current assets
C. Current assets D. None

Question#3
Assets which do not have a physical existence such as patent rights, licensing, software etc are :
A. Tangible current assets B. Tangible non current assets
C. Intangible current assets D. Intangible non current assets

Question#4
Reducing balance method recognizes depreciation:
A. usage basis B. Evenly throughout useful life
C. amount of depreciation, calculated as a D. As amount of depreciation reduces as the
fraction of remaining years and sum of life of the asset progresses
years’ digits, reduces as the life of the
asset progresses

Question#5
Property, plant and equipment can be classified as:
A. Tangible current assets B. Tangible non current assets
C. Intangible current assets D. Intangible non current assets

Question#6
Which of the following can be classified as tangible non current assets ?
A. Small tools and spare parts B. Standby generator expected to be used
for 7 years
C. A trademark D. A plot of land held for resale

Question#7
Expenditure that is made to acquire or improve long term assets that are used by the business can be
classified as :
A. Capital expenditure B. Revenue expenditure
C. Both D. None

Prepared by Maaz Imtiaz


46
Question#8
Expenditure that is made on day-to-day operating expenses can be classified as :
A. Capital expenditure B. Revenue expenditure
C. Both D. None

Question#9
Which of the following cannot be classified as tangible non-current asset:
A. A printer to be used in factory B. A printer to be used in office
C. A printer held for earning by renting to D. A printer held for resale to customers
customers

Question#10
Adil Biscuits Limited (ABL) used its own staff, assisted by contractors when required, to construct a
new factory for manufacturing chocolate biscuits.
Identify the costs listed below that cannot be capitalized .
A. A proportion of ABL’s administration B. ABL’s own staff wages for time spent
costs, based on staff time spent working on construction
C. Professional surveyor fees for managing D. Clearance of the site prior to
the construction work commencement of construction

Question#11
How the annual depreciation charge is recorded ?
A. Provision for depreciation Dr , Profit and B. Profit and loss Dr , Provision for
loss Cr depreciation Cr
C. Provision for depreciation Dr , Fixed asset D. Fixed asset Dr , Provision for depreciation
Cr Cr

Question#12
A machine was purchased for Rs1,500,000 and depreciation is calculated @15% annually by straight
line method . After 3 years it was sold for Rs450,000 . What is the amount to be included in statement
of profit or loss ?
A. Rs150,000Profit B. Rs260,000Loss
C. Rs250,000 Profit D. Rs375,000 loss

Question#13
A transport company purchased a commercial bus to travel from Karachi to Islamabad amounting to
Rs 1.5Mn. Which is the appropriate depreciation policy to be followed by the transport company to
depreciate its bus .
A. Straight line method B. Reducing balance method
C. Diminishing balance method D. B and C

Question#14
How the loss on disposal of fixed asset is recorded?
A. Profit and loss A/c Dr , Fixed asset Cr B. Profit and loss A/c Dr , Disposal A/c Cr
C. Fixed asset Dr , Profit and loss A/c Cr D. Disposal A/c Dr , Profit and loss A/c Cr

Prepared by Maaz Imtiaz


47
Question#15
A cloth manufacturer buys a machine for Rs200,0000 . It have estimated useful life of five years with a
scrap value of Rs50,000 . Depreciation is charged on straight line method .
What is rate of depreciation on annual basis ?
A. 20% B. 17%
C. 15% D. 13%

Question#16
A vehicle that originally cost for Rs1,950,000 is now sold for Rs1,200,000 . The balance on provision for
depreciation account for this vehicle is Rs780,000.
What is is the effect of this transaction ?
A. Profit in sale Rs30,000 B. Loss on sale Rs30,000
C. Profit in sale Rs750,000 D. Loss on sale Rs750,000

Question#17
Younus purchased motor vehicle for Rs1,500,000. He estimated the residual value at Rs300,000. He
decided to write-off motor vehicle over 4 years . What is the net book value after 2 years ?
A. Rs800,000 B. Rs750,000
C. Rs1,200,000 D. Rs 900,000

Question#18
What is depreciation?
A. A reserve to finance the purchase of B. An entery to write off the cost of a non
future non current assets. current asset when it is scrapped
C. A system of matching the benefits of D. A reserve to pay for repairs which
using a non current assets with its cost increase with the non current assets’ age

Question#19
What is the main purpose of charging depreciation on fixed assets?
A. To spread the cost of assets over their B. To reduce the cost of repairing assets
useful life
C. To record the assets at their market value D. To provide funds for the replacement of
assets.

Question#20
A plant costing Rs25Mn is depreciated by 25% annually on the reducing(diminishing) balance method .
What is the depreciation charge for second year?
A. Rs0.46875 B. Rs4,687,500
C. Rs46,875 D. Rs468,750

Question#21
Which entry is required to record annual depreciation charge on machinery?
A. Provision for depreciation of machinery B. Machinery at cost account Dr ; Income
account Dr ; Income statement Cr statement Cr
C. Income statement Dr ; Machinery at cost D. Income statement Dr ; Provision for

Prepared by Maaz Imtiaz


48
account Cr depreciation of machinery account Cr

Question#22
What would occur using he diminishing balance method?
A. Depreciation charge in year 2 is zero B. Depreciation charge in year 2 is same as
year 1
C. Depreciation charge in year 2 is less than D. Depreciation charge in year 2 is greater
depreciation charge in year 1 than year 1

Question#23
Mr Anas purchased machinery for Rs100,000 and depreciate it by 20% per annum usong the straight
line method . After 3 years he sold it for Rs25,000.
What was the profit or loss in sale ?
A. 25,000 loss B. 25,000 profit
C. 15,000 loss D. 15,000 profit

Question#24
A transport company purchased a commercial bus to travel from Karachi to Islamabad amounting to
Rs 1.5Mn.It is to be depreciated on the reducing balance method@25% per annum. What will be the
book value of vehicles after three years?
A. Rs600,500 B. Rs632,813
C. Rs375,000 D. Rs1,125,000

Question#25
A plant is depreciated at 15% per annum using reducing balance method . A plant is sold after 2 years
from the date of acquisition at book value. Original cost of plant was Rs200,000 .
Determine sale price ?
A. Rs140,000 B. Rs144,500
C. Rs170,000 D. Rs145,400

Question#26
An entity purchased a machinery. The invoice for the machinery showed the following items:
Rs
Machinery cost 46,000,000
Delivery cost 800,000
Warranty cost (one year covered) 1,500,000
Amount payable 48,300,000
In addition, the entity incurred Rs.2.9 million in making modifications to its factory so that the
machinery could be installed.
What should be the cost of machinery entity should recognize ?
A. Rs49.7Mn B. Rs51.2Mn
C. Rs4,970,000 D. Rs50.4Mn

Question#27
Which of the following is not a component of cost of an asset?
A. Purchase price B. Delivery cost

Prepared by Maaz Imtiaz


49
C. Installation cost D. Refundable taxes

Question#28
Which of these cost is capitalized as cost of an asset?
A. Admin overheads B. Abnormal losses
C. Professional fees D. Refundable and non-refundable taxes

Question#29
Construction of Smart super market’s new warehouse began on 1 October 2019. The following costs
were incurred on the construction:
Rs
Freehold land 4,500,000
Architect fees 150,000
Site preparation 200,000
Materials 2,250,000
Direct labor costs 1,120,000
Legal fees 50,000
General overheads 250,000
The construction was completed at 1 April 2020.
What is the amount that should be included as property, plant and equipment.
A. Rs8,170,000 B. Rs8,520,000
C. Rs8,220,000 D. Rs8,270,000

Question#30
A machine costing Rs1,500,000 was delivered incurring delivery charges of Rs49,000 . Installation
charges were Rs150,000 . What is the cost of machine ?
A. Rs1,699,000 B. Rs1,799,000
C. Rs1,650,000 D. Rs1,899,000

Question#31
A company recieved work bill from their contractor of Rs. 690,000 for construction of a new watchman
room and repair of water tank. It is estimated that 10% of total bill relates to repair work. What
amount should be capitalised and/or charged as an expense?
A. Capitalised 690,000 B. Expense 690,000
C. Capitalised 621,000 , Expense 69,000 D. Capitalised69,000, Expense 621,000

Question#32
A business purchased a machinery for Rs90,000 . 60% of the amount was paid immediately and 40% of
the amount was to be paid after a month . What is the correct journal entry ?
A. Machine Dr Rs90,000 , Bank Cr 54,000 & B. Machine Dr Rs90,000 , Bank Cr 54,000 &
Recievable Cr 36,000 Payable Cr 36,000
C. Machine Dr Rs90,000 , Bank Cr 36,000 & D. Machine Dr Rs90,000 , Bank Cr 36,000 &
Recievable Cr 54,000 Payable Cr 54,000

Question#33
A business purchased a machinery for Rs90,000 . 20% of the amount was paid in advance , 40% of the

Prepared by Maaz Imtiaz


50
amount was paid at the time of delivery , 40% of the amount was to be paid after a month . What
journal entry should be recorded at the time of delivery ?
A. Machine Dr 90,000 , Bank Cr 36,000 & B. Machine Dr 90,000 , Bank Cr 36,000 &
Advance Cr 18,000 & Payable Cr 36,000 Advance Cr 18,000 & recievable Cr 36,000
C. Machine Dr 90,000 , Bank Cr 36,000 & D. Machine Dr 90,000 , Bank Cr 36,000 &
Advance Cr 18,000 & Payable Cr 18,000 Advance Cr 36,000 & Payable Cr 36,000

Question#34
Depreciable amount means
A. Residual value – Cost of an asset B. Cost of an asset – Residual value / useful
life
C. Cost of an asset – Residual value D. Cost of an asset + Residual value

Question#35
What is the net amount an entity expects to obtain from an asset at the end of its useful life?
A. Fair value B. Present value
C. Depreciated value D. Residual value

Question#36
An helicopter engine was acquired for Rs. 65 million and has life of 38000 flying hours. The helicopter
was flown 1600 hours during the year. What amount of depreciation should be charged in profit or
loss?
A. Rs2,736,842 B. Rs2,589,542
C. Rs1,985,652 D. Rs2,128,992

Question#37
A delivery van cost Rs. 900,000. It has an expected residual value after 5 years of Rs. 90,000. If the sum
of the digits method of depreciation is used, what will be the carrying amount of the asset at the end
of Year 2?
A. 420,000 B. 414,000
C. 412,000 D. 416,000

Question#38
Hamza Limited acquired a new office building on 1 November 2016. Its initial carrying amount
consisted of:
Rs
Land 2,000
Building 10,000
Air conditioning system 4,000
16,000
The estimated lives of the building structure and air conditioning system are 25 years and 10 years
respectively.
When the air conditioning system is due for replacement, it is estimated that the old system will be
dismantled and sold for Rs. 500,000.
Depreciation is time-apportioned where appropriate. At what amount will the non-current assets be
shown in Hamza Limited’s statement of financial position as at 30 April 2017?

Prepared by Maaz Imtiaz


51
A. Rs15,620,000 B. Rs15,250,000
C. Rs15,350,000 D. Rs15,625,000

Question#39
A vehicle cost Rs300,000. The vehicle was later sold for Rs90,000 and the profit on disposal was
Rs15,000 What was the accumulated depreciation of the vehicle on disposal?
A. Rs220,000 B. Rs225,000
C. Rs195,000 D. Rs190,000

Question#40
Kamran enterprises buys a machine for Rs400,000 and depreciates it at the rate of 10% per year using
the reducing balance method What is the depreciation charge for the second year of the machine’s
use?
A. Rs36,000 B. Rs32,500
C. Rs32,000 D. Rs39,000

Question#41
An entity acquired new premises at a cost of Rs.400 million on 1 January 2017. In the period to the
year end of 31 March 2017 the following further costs were incurred.
Rs in 000
Costs of initial adaptation of the building 12,000
Relevant legal cost 2,500
Monthly Cleaning contract 3,400
Cooling system necessary for machinery 28,000
Cost of machinery 12,300
What amount should appear as the cost of premises in the entity’s statement of financial position at
31 March 2017?
A. Rs418,550,000 B. Rs415,500,000
C. Rs415,550,000 D. Rs455,500,000

Question#42
A company uses the straight line method of depreciation for all its non-current assets. On 1 January,
the company bought machinery on hire purchase. The cash price was Rs200, 000 and the interest for
the year is Rs20,000. The estimated useful life of the machinery is five years with no residual value.
What is the charge for depreciation for the year ended 31 December?
A. Rs44,000 B. Rs40,000
C. Rs45,000 D. Rs50,000

Question#43
A car was part exchanged for a new car. The value placed on the old vehicle was Rs540, 000. Which
entries record the Rs540, 000 part exchange?
A. Dr Disposals account , Cr Motor Vehicles B. Dr Motor Vehicles account , Cr Cash
account account
C. Dr Motor Vehicles account , Cr Disposals D. Dr Cash account , Cr Motor Vehicles
account account

Prepared by Maaz Imtiaz


52
Question#44
An asset cost Rs1,200,600 and has accumulated depreciation of Rs840,000. The asset is sold for
Rs590,000. What is the loss or profit on disposal?
A. Rs230,000 Profit B. Rs230,000 loss
C. Rs950,000 Profit D. Rs950,000 loss

Question#45
On 1 March 2019 Jasmine Limited (JL) acquired a machine from Plant under the following terms:
Rs in 000
List price of machine 82,000
Import duty 1,500
Delivery fees 2,050
Electrical installation cost 9,500
Pre-Production testing 4,900
Purchase of a five-year maintenance contract 7,000
with Plant
In addition to the above information ML was granted a trade discount of 8% on the initial list price of
the asset and a settlement discount of 4% on remaining amount if payment for the machine was
received within one month of purchase. JL expected and paid for the plant on 25 March 2019.
On what amount, the plant should be initially measured on acquisition?
A. Rs88,372,400 B. Rs90,372,400
C. Rs89,560,000 D. Rs90,580,000

Question#46
A business sold a non-current asset. The following information is known.
Original Cost 600,000
Accumulated depreciation on date of sale 240,000
Profit on sale 70,000
What is the proceed from the sale of non-current asset?
A. Rs420,000 B. Rs430,000
C. Rs490,000 D. Rs450,000

Question#47
An asset, which would not normally be depreciated, is
A. plant and machinery B. patents
C. land D. building

Question#48
The table shows information relating to a company’s non-current assets.
Rs
Cost at 1 January 2012 100,500
Accumulated depreciation at 1 January 40,900
2012 Purchases for the year ended 31 20,500
December 2012 7,000
Disposals for the year ended 31 December 2012

Prepared by Maaz Imtiaz


53
Depreciation is 25% per annum on the reducing balance basis.
What is the depreciation charge for the year?

A. Rs21,772 B. Rs38,725
C. Rs20,450 D. Rs18,275

Question#49
On 1 October 2011 a company purchased machinery for Rs270,000. It was decided to depreciate the
asset using the reducing balance method at a rate of 20% per annum. On 30 September 2013 the asset
was sold for Rs120,000. What is the profit or loss on disposal?
A. Rs55,920 B. Rs52,800
C. Rs165,900 D. Rs160,800

Question#50
Which asset does not need to be depreciated?
A. a revalued property B. a quarry
C. an oil well D. land

Prepared by Maaz Imtiaz


54
Chapter 7 IAS-2 Inventories
Question#1
Inventories are measured at :
A. lower of cost and net realizable value B. lower of cost or net realizable value (NRV)
(NRV)
C. Both A and B D. None

Question#2
Generally for these reasons entity maintain inventories :
i. Meeting variation in production demand;
ii. Economies of scale in procurement;
iii. Guarding against price increases and stock outs.
Which of these statements are correct ?
A. Only i) is correct B. ii) and iii) are correct
C. i) and iii) correct D. i) , ii) and iii) are correct

Question#3
Inventories which are in the process of production for finished goods is called ;
A. Finished goods B. Work in process
C. Raw materials D. Both B and C

Question#4
Which of the following will become part of cost of purchase of inventory ?
1) Non-refundable import duties
2) Refundable sales tax
3) Transport and handling costs
4) Discounts and rebates
5) Other costs directly attributable to acquisition
A. 1 , 2 , 3 ,5 B. 2 ,3 ,4 , 5
C. 1 , 3 , 5 D. All of them

Question#5
A company imported raw material for use in production of goods with following details:
Rs
(000)
Invoice value (List price Rs. 1,060,000; trade discount Rs. 40,000) 1,020
Custom duty 170
Federal excise duty 60
Non adjustable income tax 265
Refundable sales tax 55
Carriage 115
Commission paid to agent for clearance of raw materials 1,600
Calculate the cost of purchase on inventories.
A. Rs3,230,000 B. Rs3,280,000

Prepared by Maaz Imtiaz


55
C. Rs3,285,000 D. Rs3,175,000

Question#6
Which of the following combination of examples regarding fixed cost is correct ?
A. Depreciation , Admin Expenses , Indirect B. Depreciation , Admin Expenses ,
labour Management cost
C. Depreciation , Admin Expenses , Indirect D. Depreciation , Admin Expenses , Direct
labour , Direct labour labour

Question#7
Which of the following costs are included in conversion costs?
A. Factory supervisor’s wages B. Carriage in
C. Carriage outwards D. Commission of selling staff

Question#8
After preparing draft accounts, Adnan reviews his closing inventory. He discovers that some items
included at cost of Rs. 2,850 can be sold for Rs. 2,790 after incurring selling costs of Rs. 95. What effect
will any required adjustment have on Adnan’s profits?
A. Profit decreases by 155 B. Profit decreases by 95
C. Profit decreases by 195 D. Profit increases by 155

Question#9
Habib Corporation sells three products – A, B and C. The following information was available at the
yearend:
A B C
Original cost ( Rs/unit) 15 13 15
Estimated selling price (ESP) 18 15 13
Selling and distribution costs 2 3 2

Units Units Units


Inventory: units held 350 360 250
The value of inventory at the end of year should be?
A. Rs14,030 B. Rs12,670
C. Rs13,030 D. Rs12,320

Question#10
The closing stock of Daniyal amounts to Rs. 140,400. But later on it was discovered that some
damaged items were included having cost of Rs. 30,000. Total repair cost is expected to be Rs. 7,500.
After repair these could be sold for Rs. 29,000. What is the correct value of Daniel inventory?
A. Rs119,700 B. Rs131,900
C. Rs124,800 D. Rs128,500

Question#11
Which of the following is not allowed to be included as a cost of inventory?
A. Fixed manufacturing overheads B. Shipping
C. Storage costs D. Non-recoverable taxes

Prepared by Maaz Imtiaz


56
Question#12
Which of the following is allowed as a cost of inventory?
A. Variable manufacturing overheads B. Selling costs
C. Storage costs D. Abnormal waste

Question#13
Which of the following statements define the term ‘net realizable value’?
A. The estimated selling price in the ordinary B. The estimated selling price in the ordinary
course of business plus the estimated course of business less the estimated
costs of completion less the estimated costs of completion and the estimated
costs necessary to make the sale costs necessary to make the sale
C. The estimated cost of completion less D. The estimated costs necessary to make
selling price in the ordinary course of the sale plus selling price in the ordinary
business course of business less the estimated
costs of completion

Question#14
Normal capacity is the production expected to be achieved on average over a number of periods or
seasons under normal circumstances, taking into account the loss of capacity resulting from planned
maintenance.
A. True B. False
C. Either A or B D. None

Question#15
The actual level of production may be used if it __________ normal capacity.
A. equals B. approximates
C. exceeds D. is less than

Question#16
Spurce Limited, imported raw materials from Japan worth Rs.11 million. They paid Rs. 850,000 as
import duties and Rs. 150,000 as import taxes (the import taxes were subsequently refunded by the
government). They paid Rs. 200,000 for transportation of the materials from Japan and another Rs.
250,000 as port handling charges for loading the materials at Japan. Marketing expenses were Rs.
80,000 and the general administrative overheads amounted to Rs. 170,000.
What will be the value of inventories?
A. Rs11,150,500 B. Rs12,300,300
C. Rs11,150,000 D. Rs12,300,000

Question#17
An entity sold goods of worth Rs.1.2 million, the manufacturing cost of the goods were Rs. 550,000.
The carriage outwards are Rs. 60,000 and commission paid to agent were also Rs. 60,000. What is the
gross and net profit?
A. Gross profit = 650,000 and net profit = B. Gross profit = 530,000 and net profit =
530,000 650,000
C. Gross profit = 600,000 and net profit = D. Gross profit = 600,000 and net profit =

Prepared by Maaz Imtiaz


57
500,000 530,000

Question#18
The following information is related to a laptop dealer about his inventory at year end.
Laptop Cost value (Rs.) Net realisable value (Rs.)
A 50,000 30,300
B 130,000 130,500
C 140,200 130,900
D 140,900 150,000
What value of inventory should be shown in his Statement of Financial Position prepared at the year
end?
A. Rs432,100 B. Rs452,100
C. Rs441,200 D. Rs460,900

Question#19
Any amount of write-down of inventories to net realisable value should?
A. Recognized as a current liability in the B. Recognised as an expense in the
statement of financial position subsequent period in which such write-
down is warranted
C. Recognised as an expense in the period in D. Treated as a deferred expense and
which the write-down occurs written off based on the average
inventory holding period

Question#20
Affan is a computer dealer. His inventory includes a CPU which cost Rs. 158,000. Affan expects to
spend Rs. 7,000 on repairing the CPU which will mean that he will be able to sell it for Rs. 260,000. To
replace the same item of inventory would cost Rs. 255,000. At what value should the CPU be included
in Affan’s inventory?
A. Rs152,600 B. Rs151,500
C. Rs153,500 D. Rs158,000

Question#21
What is impact on closing inventory if an item having cost of Rs. 3,500 and a net realizable value of Rs.
4,000 has been omitted from year - end inventory count?
A. Understated by 3,500 B. Understated by 4,000
C. Overstated by 3,500 D. Overstated by 4,000

Question#22
The estimated selling price in the ordinary course of business less estimated cost of completion and
estimated cost of sale is called
A. Current value B. Net realisable value
C. Fair value D. Market value

Question#23
Hamdani Associates has 40 units of inventory, out of which 10 units are damaged. The cost per unit is
Rs. 2,000 and normal selling price is Rs. 3,000. The damaged units are expected to be sold at 80% of

Prepared by Maaz Imtiaz


58
normal selling price. The selling cost of Rs. 100 are incurred on each unit sold, whether normal or
damaged. What is the amount of write down of inventory, if any?
A. Rs. Nil B. Rs. 80,000
C. Rs. 6,000 D. Rs. 600

Question#24
The following information relates to Abdullah Enterprise (AE) for the month of September 2019:
1 September Opening inventory of 400 units @ Rs. 100 each = Rs. 40,000
10 Purchased 100 units @ Rs. 150 each = Rs. 15,000
September
25 Purchased 300 units @ Rs. 200 each = Rs. 60,000
September
No units were sold during September 2019. What would be cost of closing inventory per unit of AE as
at 30 September 2019 valued on weighted average (perpetual)?
A. Rs. 187.5 per unit B. Rs. 150 per unit
C. Rs. 143.75 per unit D. Rs. 110 per unit

Question#25
Which of the following items should be disclosed as per the requirements of IAS 2?
A. Average lead time of procurement for B. Amount of expense recognised due to
major classes of inventories write down of inventories
C. List of major customers to whom the D. Average holding period of inventories of
inventories were sold during the the entity as at the end of the reporting
reporting period period

Question#26
Meta Building Materials used 300 cement bags from inventory for constructing parking area of their
office building. How these 300 cement bags should be accounted for under periodic inventory
recording system?
A. Debit Inventory & Credit Non-current B. Debit Non-Current assets & Credit
assets Purchases
C. Debit Non-Current assets & Credit D. Debit Inventory & Credit Purchases
Inventory

Question#27
Hamdani Associates sold a generator to Ammar Enterprises for Rs. 1,880,000. This price is net of Rs.
80,000 special discount. Jaffer Associates normally sells items at 25% mark-up and uses perpetual
inventory system to record its inventory. Which of the following entry is correct to update the
inventory?
A. Debit Inventory Rs. 1,550,000 & Credit B. Debit Cost of Sales Rs. 1,105,000 & Credit
Cost of sales Rs. 1,550,000 Inventory Rs. 1,105,000
C. Debit Cost of Sales Rs. 1,568,000 & Credit D. Debit Cost of Sales Rs. 1,162,000 & Credit
Inventory Rs. 1,568,000 Inventory Rs. 1,162,000

Question#28
Which one of the following are recognized as expense under IAS 2?

Prepared by Maaz Imtiaz


59
A. Inventory remained unsold at the end of B. Amount of write down to NRV
period
C. Inventory pledged with the bank as D. None
security for the loan financing

Question#29
Which of the following costs must be expensed?
A. Distribution cost B. Variable production overheads that are
allocated to each unit based on actual
usage
C. Import duties on raw materials that are D. Costs of purchase that are paid to the
paid to the authorities suppliers of raw materials

Question#30
Abdullah Enterprises uses perpetual inventory system, which of the following TWO are correct for
recoding a credit sales transaction?
A. No entry B. Debit Cost of sales & Credit Inventory
C. Debit Cost of sales & Credit Purchases D. Debit Receivables & Credit Sales

Question#31
Which of the following items cannot be included in the cost of inventories?
A. Variable production overheads B. Fixed production overheads
C. The cost of abnormal wastage of D. Irrecoverable import duties payable on
materials and labour the acquisition of inventories

Question#32
The cost formulas permitted by IAS2 are:
A. FIFO and LIFO B. FIFO and AVCO
C. FIFO, LIFO and AVCO D. LIFO and AVCO

Question#33
The net realisable value of inventories is defined by IAS2 as:
A. Cost price B. Selling price less costs of completion and
selling costs
C. Selling price D. Selling price less costs of completion

Question#34
Which of the following items should be included in the cost of inventories?
A. The cost of abnormal wastage of B. Selling costs
materials and labour
C. The cost of storing finished goods D. Conversion costs

Question#35
The FIFO cost formula assumes that:
A. Newer inventory items are sold or B. The inventory items which are sold or

Prepared by Maaz Imtiaz


60
consumed before older inventory items consumed are a mixture of those acquired
previously
C. The inventory items which are sold or D. The inventory items which are sold or
consumed are those acquired most consumed are those acquired longest ago
recently

Question#36
On 31 December 2015, a company has partly-completed inventory with a cost to date of Rs26,300. It is
expected that further costs of Rs8,900 will be incurred in order to complete the inventory. It will then
be sold for Rs47,500. Selling costs will be Rs2,000.
The cost and the net realisable value of this inventory at 31 December 2015 are:
A. Rs26,300 and Rs38,600 B. Rs35,200 and Rs47,500
C. Rs26,300 and Rs36,600 D. Rs35,200 and Rs45,500

Question#37
IAS2 states that inventories should be measured at:
A. Cost B. The lower of cost and net realisable value
C. The higher of cost and net realisable value D. Net realisable value

Question#38
If production is abnormally low, the amount of fixed overheads allocated to each unit of production
should be calculated by dividing total fixed overheads by the number of units produced.
A. True B. False
C. Either A or B D. None

Question#39
At the end of an accounting period, the cost of a company's inventory is Rs450,000. This includes
damaged items with a cost of Rs25,000 which are expected to be sold for only Rs10,000 (less selling
expenses of 5%). All other items of inventory have a net realisable value which exceeds cost.
The amount at which the company's inventory should be recognised at the end of the period is:
A. Rs425,000 B. Rs450,000
C. Rs435,000 D. Rs434,500

Question#40
A company's inventories should be measured at the lower of total cost and total net realisable value.
A. True B. False
C. Either A or B D. None

Question#41
A company uses periodic inventory system, which of the following TWO are correct for recoding a
credit sales return transaction?
A. Debit Cost of sales & Credit Inventory B. Debit Sales Return & Credit Receivables
C. Debit Inventory & Credit Cost of sales D. No entry

Question#42
If trial balance includes “purchase” and “purchase return” account, it is an indication of:

Prepared by Maaz Imtiaz


61
A. Periodic inventory recording system B. Perpetual inventory recording system
C. FIFO method D. Weighted average method

Question#43
What is correct entry for goods taken by owner for personal use?
A. Cr Sales account and Dr Drawings account B. Cr Trading account and Dr Drawings
with the sale price of the goods. account with the selling price of the
goods.
C. Cr Opening Inventory account and Dr D. Cr Purchases account and Dr Drawings
Drawings account with cost price of the account with the cost price of the goods.
goods.

Question#44
A company which makes only one type of product incurs fixed production overheads of Rs180,000 for
an accounting year. Actual production during the year was 30,000 units. Normal production is 24,000
units per annum.
The amount of fixed production overheads that should be allocated to each unit of production is:
A. Rs Nil B. Rs7.50
C. Rs6 D. Rs30

Question#45
Maaz had opening inventory of Rs. 1,600,000. Purchases made during the period were Rs. 2,650,000.
Sales during the period were Rs. 4,600,000 and he had closing inventory of Rs. 900,000. Gross profit for
the period was?
A. Rs. 1,350,000 B. Rs. 1,550,000
C. Rs. 1,250,000 D. Rs. 1,650,000

Question#46
Which of the following is NOT a disclosure requirement of IAS 2?
A. Location of each place where entity keeps B. The amount of any write-down of
its inventory inventories recognised as an expense in
the period
C. The cost formula used D. Which of the following is NOT a disclosure
requirement of IAS 2?

Question#47
Which of the following cost models is not permitted under IAS 2?
A. Actual cost B. Weighted Average
C. Last in, First out (‘LIFO’) D. First in, First out (‘FIFO’)

Question#48
An entity had opening inventory of 36,000 units @Rs. 3.9 per unit. During the month it made
purchases of 41,000 units @Rs. 4.5 per unit. Sales were 60,000 units.
What is value of cost of goods sold during the month if the entity uses continuous weighted average
method for inventory valuation?
A. Rs254,900 B. Rs253,200

Prepared by Maaz Imtiaz


62
C. Rs253,500 D. Rs254,500

Question#49
A company uses periodic inventory system, which of the following TWO are correct for recoding a
credit sales transaction?
A. Debit Cost of sales & Credit Inventory B. Debit Receivables & Credit Sales
C. Debit Cost of sales & Credit Purchases D. No entry is required

Question#50
Yousuf flour mills gave 1,600 bags of flour to poor community as a donation. What is correct journal
entry to record this transaction under perpetual inventory method?
A. Debit Drawings & Credit Purchases B. Debit Expenses & Credit Purchases
C. No entry D. Debit Expenses & Credit Inventory

Prepared by Maaz Imtiaz


63
Chapter 8 Accounting for Manufacturing
Question#1
Which is a factory overhead ?
A. Carriage on raw materials B. Cost of raw materials
C. Production supervisor’s wages D. Wages of machine operators

Question#2
How is cost of production is calculated ?
A. Prime cost less factory overheads B. Prime cost plus factory overheads
C. Prime cost less factory overheads less D. Prime cost plus factory overheads less
increase in work in progress increase in work in progress

Question#3
Which one of them is prime cost ?
A. Sum of direct material and direct labour B. Sum of indirect material and direct labour
costs costs
C. Sum of direct material and selling costs D. Sum of direct material and factory
overhead costs

Question#4
A restaurant incurs following cost ?
1) wages of the cooking staff
2) depreciation of cooking equipment
3) costs of ingredients for meals
4) rent paid for the restaurant shop
What are indirect costs for an each meal?
A. 1, 2 and 3 only B. 1, 2, 3 and 4
C. 2 and 4 only D. 1 and 2 only

Question#5
Which one of the following would be included as direct labour?
A. Human resource manager in B. Maintenance manager in a company
departmental store producing bikes
C. Cook at fast food company D. General manager in a cars company

Question#6
Which of the following is most likely to be treated as an indirect cost by a house construction
contractor?
A. Bricks B. Windows
C. Doors and locks D. Glue and adhesives

Question#7
Direct costs are the costs that
A. Are directly charged to department B. Are under the control of managment of

Prepared by Maaz Imtiaz


64
the company
C. Can be directly identified with a product D. Are directly under the control of a
of service manager

Question#8
How is the factory cost of production is calculated ?
A. Direct material + Direct labour + Direct B. Direct material + Direct labour + Direct
expenses + Total overheads expenses + Factory overheads
C. Direct material + Direct labour + Direct D. Direct material + Direct labour
expenses

Question#9
Which TWO of the following are direct costs for a confectionery producing business ?
A. Butter purchased for cakes B. Hours of cake decorator’s time
C. Rent of shop where confectionery items D. Electricity bill of the shop where
are sold confectionery items are sold

Question#10
Which TWO of the following are indirect costs for a business that produces cushion covers?
A. Foam used in cushion covers B. Surf purchased to wash fabric before
production
C. Lubricant purchased for sewing machines D. white cotton fabric purchased

Question#11
Which of the following costs would be classified as an indirect cost?
A. Steel for making cars B. Rubber for making tyres
C. Tyres installed in car by car manufacturer D. Depreciation of plant used in production
of vehicles

Question#12
Which is a direct cost ?
A. Supervisor’s wages B. Raw materials used
C. Lightening and heating D. Depreciation of machinery

Question#13
A manufacturing business has purchased material costing Rs. 214,000 on credit during the month of
May 2021. At the start of March, there were inventories of raw material of Rs. 25,000. During the
month Rs. 188,000 of direct materials and Rs. 35,000 of indirect materials were issued for production.
What is correct accounting entry to record issue of indirect materials?
A. Dr. Inventory (WIP) Rs. 35,000 and Cr. B. Dr. Production Overheads Rs. 35,000 and
Inventory (Materials) Rs. 35,000 Cr. Inventory (Materials) Rs. 35,000
C. Dr. Inventory (Materials) Rs. 35,000 and D. Dr. Inventory (Materials) Rs. 35,000 and
Cr. Inventory (WIP) Rs. 35,000 Cr. Production Overheads Rs. 35,000

Prepared by Maaz Imtiaz


65
Question#14
During the month of July 2020, a manufacturing business issued Rs. 20.2 million of direct materials to
the factory and Rs. 4.9 million of indirect materials. What is the double entry for these issues of
materials?
A. Dr. Materials Control Rs. 25.1m & Cr. WIP B. Dr. WIP Rs. 4.9m & Dr. Production OH Rs.
Rs. 4.9m & Cr. Production OH Rs. 20.2m 20.2m & Cr. Materials Control Rs. 25.1m
C. Dr. WIP Rs. 20.2m & Dr. Production OH D. Dr. Materials Control Rs. 25.1m & Cr. WIP
Rs. 4.9m & Cr. Materials Control Rs. Rs. 20.2m & Cr. Production OH Rs. 4.9m
25.1m

Question#15
Which group only contains indirect cost ?
A. Carriage inwards , factory rent , wages of B. Carriage inwards , raw materials , wages
factory supervisors of machine operators
C. Depreciation of machinery , factory rent , D. Depreciation of machinery , raw materials
wages of factory supervisors , wages of machine operators

Question#16
During the month of March 2020, a manufacturing business payroll totalled Rs. 26.8 million. Rs. 5.2
million of this amount was for indirect wages and the remainder was for direct worker wages.
What is the double entry for the above costs?
A. Dr. WIP Rs. 5.2 m & Dr. Production OH Rs. B. Dr. WIP Rs. 21.6m & Dr. Production OH
21.6m & Cr. Wages Control Rs. 26.8m Rs. 5.2m & Cr. Wages Control Rs. 26.8m
C. Dr. Wages Control Rs. 26.8m & Dr. WIP D. Dr. Wages Control Rs. 26.8m & Cr. WIP Rs.
Rs. 5.2m & Cr. Production OH Rs. 21.6m 21.6m & Cr. Production OH Rs. 5.2m

Question#17
During month, the direct production workers worked for 810 hours including 190 hours of general
overtime. The indirect workers worked for 230 hours, which included 38 hours of general overtime.
The direct production workers have a basis rate of Rs. 790 per hour and the indirect have a basic rate
of Rs. 590 per hour.
A. Rs639,000 B. Rs790,000
C. Rs775,600 D. Rs890,445

Question#18
What entry should be made in the cost accounting system on sale of finished goods?
A. Debit Cost of sales (P&L) & Credit Finished B. Debit Inventory (WIP) & Credit Finished
goods goods
C. Debit Finished goods & Credit Inventory D. Debit Finished goods & Credit Cost of
(WIP) sales (P&L)

Question#19
How is prime cost calculated?
A. Direct materials + direct labour B. Direct materials + direct labour + direct
expenses
C. Direct materials + direct labour + direct D. Direct materials + direct labour + direct

Prepared by Maaz Imtiaz


66
expenses + factory overheads expenses + factory overheads + change in
work in progress

Question#20
A manufacturing business has purchased material costing Rs. 214,000 on credit during the month of
May 2021. At the start of March, there were inventories of raw material of Rs. 25,000. During the
month Rs. 188,000 of direct materials and Rs. 35,000 of indirect materials were issued for production.
What is correct accounting entry to record issue of direct materials?
A. Dr. Inventory (WIP) Rs. 188,000 and Cr. B. Dr. Production Overheads Rs. 188,000 and
Inventory (Materials) Rs. 188,000 Cr. Inventory (Materials) Rs. 188,000
C. Dr. Inventory (Materials) Rs. 188,000 and D. Dr. Inventory (Materials) Rs. 188,000 and
Cr. Inventory (WIP) Rs. 188,000 Cr. Production Overheads Rs. 188,000

Question#21
Which of the following account is most unlikely to be part of factory ledger?
A. Direct Labour (wages control) B. Plant and Machinery (installed in factory
premises)
C. Production overheads D. Inventory (Work in progress)

Question#22
A business has an overhead absorption rate of Rs. 510 per machine hour, based on a budgeted activity
level of 11,990 hours. In the period covered by the budget, actual machine hours worked were 3%
more than the budgeted hours and actual overhead expenditure incurred was Rs. 5,648,900.
What was the budgeted overheads for the period?
A. Rs6,114,000 B. Rs5,114,000
C. Rs6,114,900 D. Rs5,270,000

Question#23
A business has an overhead absorption rate of Rs. 510 per machine hour, based on a budgeted activity
level of 11,990 hours. In the period covered by the budget, actual machine hours worked were 3%
more than the budgeted hours and actual overhead expenditure incurred was Rs. 5,648,900.
What was the total absorbed overheads for the period?
A. Rs6,298,347 B. Rs5,298,300
C. Rs5,598,347 D. Rs. 5,375,400

Question#24
A business has an overhead absorption rate of Rs. 510 per machine hour, based on a budgeted activity
level of 11,990 hours. In the period covered by the budget, actual machine hours worked were 3%
more than the budgeted hours and actual overhead expenditure incurred was Rs. 5,648,900.
What was the total over or under absorption of overheads in the period?
A. Rs.647,447 overabsorbed B. Rs.649,447 underabsorbed
C. Rs.647,447 underabsorbed D. Rs.649,447 overabsorbed

Question#25
A manufacturer provides the following information
Rs

Prepared by Maaz Imtiaz


67
Factory wages 29,000
Carriage on raw materials 2,500
Machine repairs 5,000
Supervisor’s wages 9,500
Raw materials used 30,000
What is the prime cost of manufacturing?
A. Rs61,500 B. Rs71,000
C. Rs76,000 D. Rs73,500

Question#26
A manufacturer provides the following information for his year end .
Rs
Cost of goods manufactured 520,000
Inventory (Finished goods) Opening 245,000
Inventory (Finished goods) Closing 150,000
Calculate the Cost of goods sold during the year.
A. Rs615,000 B. Rs915,000
C. Rs765,000 D. Rs625,000

Question#27
Total manufacturing costs incurred include:
A. direct labor costs. B. raw materials costs.
C. manufacturing overhead. D. All of the above

Question#28
A manufacturer had the following cost
Rs
Raw materials 8,000
Wages of factory workers 4,000
Wages of factory supervisor’s 1,000
Wages of office workers 2,000
Fixed factory overheads 4,500
Fixed office overheads 1,500
What is the factory cost of production ?
A. Rs19,500 B. Rs17,500
C. Rs21,000 D. Rs20,900

Question#29
Hamza Enterprises , a manufacturer provides the following information at the year end
Rs
Prime Cost 550,000
Production overheads 190,000
Selling overheads 85,000
Administrative overheads 75,000
WIP inventory at start and end of the year Nil
Calculate the Cost of goods manufactured during the year.

Prepared by Maaz Imtiaz


68
A. Rs815,000 B. Rs740,000
C. Rs710,000 D. Rs900,000

Question#30
Hamza Enterprises , a manufacturer provides the following information at the year end
Rs
Prime Cost 550,000
Production overheads 190,000
Selling overheads 85,000
Administrative overheads 75,000
WIP inventory at start and end of the year 35,000
Calculate the Cost of goods manufactured during the year.
A. Rs815,000 B. Rs740,000
C. Rs710,000 D. Rs900,000

Question#31
A manufacturer provides the following information
Rs
Direct expenses 7,000
Direct material 21,000
Direct labour 16,000
Office expenses 5,000
Factory heating and lighting 11,000
Selling expenses 9,000
What is the prime cost of production ?
A. Rs64,000 B. Rs55,000
C. Rs70,000 D. Rs44,000

Question#32
Aadil Limited , a manufacturer prepares manufacturing account for its year end and provides following
information
Rs
Cost of direct material consumed 370,000
Direct labour wages 55,000
Depreciation (80% relates to production) 25,000
Royalty paid on number of units of product produced 22,500
Rent and utilities expenses of factory 12,500
Calculate the prime cost ?
A. Rs467,500 B. Rs447,500
C. Rs452,500 D. Rs472,500

Question#33
Aadil Limited , a manufacturer prepares manufacturing account for its year end and provides following
information
Rs
Cost of direct material consumed 370,000

Prepared by Maaz Imtiaz


69
Indirect labour wages and indirect materials 17,000
Depreciation (80% relates to production) 25,000
Royalty paid on number of units of product produced 22,500
Rent and utilities expenses of factory 12,500
Calculate the production overheads for the year.
A. Rs48,000 B. Rs49,000
C. Rs48,500 D. Rs49,500

Question#34
Aadil Limited , a manufacturer prepares manufacturing account for its year end and provides following
information
Rs
Cost of direct material consumed 370,000
Indirect labour wages and indirect materials 17,000
Depreciation (80% relates to production) 25,000
Royalty paid on number of units of product produced 22,500
Rent and utilities expenses of factory 12,500
Direct labour 55,000
Calculate the production overheads for the year.
A. Rs497,500 B. Rs497,000
C. 597,500 D. Rs597,000

Question#35
Nails,depreciation,and foreman salaries are examples of:
A. Direct labour B. Manufacturing overhead
C. raw materials D. None

Question#36
A business provides the following information
Rs
Prime cost 290,000
Factory overheads 120,000
Closing work in progress 18,000
What is cost of production?
A. Rs392,000 B. Rs428,000
C. Rs410,000 D. Rs395,000

Question#37
Lumber used in construction of a building is part of:
A. raw material costs. B. Labour cost
C. Manufacturing overhead D. None

Question#38
Direct labor includes the wages of:
A. the shop foreman. B. an hourly worker producing the product.

Prepared by Maaz Imtiaz


70
C. maintenance workers. D. administrators.

Question#39
A entity engaged in manufacturing has policy of holding no inventory of any type whether raw
materials, WIP or finished goods. During an accounting period it only produced goods from raw
material purchased by it. Its cost of sales shall be equal to:
A. Prime Cost + Production Overheads B. Direct labour + Production Overheads
C. Prime Cost D. Cost of direct materials consumed

Question#40
Adil Limited is manufacturer of hand bags. Following figures have been provided for the month of
March 2020:
Rs
Raw materials 420,000
Direct labour cost 290,000
Selling and administrative salaries 145,000
Rent for factory 115,000
Depreciation - Factory equipment 75,000
Indirect labour cost 42,000
Factory utilities 25,000
Factory insurance 15,000
Only 50% of the utilities expenses and 50% of the insurance expense apply to factory operations, the
remaining amount should be charged to selling and administrative expenses.
What is the amount of factory overheads for the month?
A. Rs962,000 B. RS692,000
C. RS272,000 D. Rs252,000

Question#41
Manufacturing overhead includes all manufacturing costs:
A. including raw materials. B. including overhead.
C. excluding raw materials and direct labor D. None

Question#42
Which of the following shows over-absorbed overheads?
A. Absorbed overheads > Actual overheads B. Actual overheads > Absorbed overheads
C. Budgeted overheads > Actual overheads D. Actual overheads > Budgeted overheads

Question#43
Which of the followings shows “Absorbed overheads”?
A. Normal capacity x Actual rate B. Actual production x Pre-determined rate
C. Budgeted production x Actual rate D. Budgeted production x Pre-determined
rate

Question#44
The following data relates to Spruce Limited who is in process of preparing manufacturing account for
the year ended June 30, 2019.

Prepared by Maaz Imtiaz


71
Rs
Inventory (Materials) 1 July 2018 76,000
Inventory (Material) 30 June 2019 110,000
Purchases of direct materials during the year 425,000
Return outwards during the year 27,000
Calculate the cost of materials available for consumption.
A. Rs474,000 B. Rs475,550
C. Rs355,410 D. Rs365,800

Question#45
The following data relates to Spruce Limited who is in process of preparing manufacturing account for
the year ended June 30, 2019.
Rs
Inventory (Materials) 1 July 2018 76,000
Inventory (Material) 30 June 2019 110,000
Purchases of direct materials during the year 425,000
Return outwards during the year 27,000
Calculate the cost of materials consumed during the year.
A. Rs364,000 B. Rs364,400
C. Rs465,800 D. Rs455,900

Question#46
Which one of the following equation is correct for calculating cost of goods manufactured?
A. Factory Cost + Opening inventory (WIP) – B. Factory Cost + Opening inventory (raw
Closing inventory (WIP) material) – Closing inventory (raw
material)
C. =Prime Cost + Opening inventory (WIP) – D. Prime Cost + Opening inventory (raw
Closing inventory (WIP) material) – Closing inventory (raw
material)

Question#47
What entry is made for over absorbed overheads at the end of period?
A. Debit Production Overheads & Credit Cost B. Debit Cost of sales (P&L) & Credit
of sales (P&L) Production Overheads
C. Debit Inventory (WIP) & Credit Production D. Debit Production Overheads & Credit
Overheads Cash / Accrual

Question#48
What entry should be made for under absorbed overheads at the end of period?
A. Debit Production Overheads & Credit Cost B. Debit Cost of sales (P&L) & Credit
of sales (P&L) Production Overheads
C. Debit Inventory (WIP) & Credit Production D. Debit Production Overheads & Credit
Overheads Cash / Accrual

Question#49
What entry is made when overheads are absorbed in production?

Prepared by Maaz Imtiaz


72
A. Debit Production Overheads & Credit Cost B. Debit Cost of sales (P&L) & Credit
of sales (P&L) Production Overheads
C. Debit Inventory (WIP) & Credit Production D. Debit Production Overheads & Credit
Overhead Cash / Accrual

Question#50
A manufacturer has provided following information
Rs
Raw materials consumed 450,000
Direct labour cost 300,000
Indirect labour 50,000
Calculate prime cost ?
A. Rs750,000 B. Rs800,000
C. Rs700,000 D. Rs850,000

Prepared by Maaz Imtiaz


73
Chapter 9 Preparation of Financial Statements
Question#1
Financial statements should present _______ the financial position, financial performance and cash
flows of the entity.
A. Truly B. Fairly
C. Appropriately D. Correctly

Question#2
If liablity meets any of the following criteria:
 The entity expects to settle the liability in its normal operating cycle.
 The liability is held primarily for the purpose of trading. This means that all trade payables are
current liabilities, even if settlement is not due for over 12 months after the end of the
reporting period.
 It is due to be settled within 12 months after the end of the reporting period.
 The entity does not have the unconditional right to defer settlement of the liability for at least
12 months after the end of the reporting period.
It shall be classified as ?
A. Liablity B. Current liablity
C. Non current liablity D. None of them

Question#3
If a company obtains a five-year bank loan, where none of the loan principal is repayable until the end
of the loan period .
What is status of loan principal at year 4 ?
A. Current liablity B. Non current liablity
C. Current asset D. Non current asset

Question#4
Which is not the part of set of financial statement
A. a statement of changes in equity B. Aging analysis of inventory
C. a statement of comprehensive income D. a statement of financial position

Question#5
Adnan has prepared his trial balance for the year ended 31 December 2019. He has provided following
information relating to drawings:
Rs Debit
Drawings 300,000
Adnan’s son works as the head of administration and received a salary of Rs. 15 thousand per month,
which has been included in drawings.
What is the amount of drawings to be shown in the statement of financial position?
A. Rs120,000 B. Rs130,000
C. RS120,500 D. Rs285,000

Prepared by Maaz Imtiaz


74
Question#6
Following is the trial balance of Adnan for the year ended 31 December 2019
Rs
Insurance 1,200,000
Insurance is prepaid to the extent of Rs. 770,000.
What is the amount of insurance premium to be shown in the statement of comprehensive income
and statement of financial position?
A. Insurance expense Rs. 1,200,000; Prepaid B. Insurance expense Rs.1,970,000; Prepaid
insurance Rs. 770,000 insurance Rs. 770,000
C. Insurance expense Rs. 650,000; Prepaid D. Insurance expense Rs. 430,000; Prepaid
insurance Rs. 1,200,000 insurance Rs. 770,000

Question#7
Akram is in process of preparation of trial balance for the year ended 31 December 2020.
Rs Debit
Purchases 225,000
Akram withdrew goods costing Rs. 125,000 for personal use during the year. However, no entry was
made to record the withdrawal of goods.
What is the amount of purchases to be shown in statement of comprehensive income?
A. Rs100,000 B. Rs225,000
C. Rs125,000 D. Rs350,000

Question#8
Following is the trial balance of Adnan for the year ended 31 December 2020:
Rs in 000 Rs in 000
Debit Credit
Trade discount 240 Sales 36,260
Sales return 1,750
Discount allowed 475
Assistant account has informed that sales was recorded incorrectly on gross amount and trade
discount was debited to correct it.
What is the amount of sales to be shown in statement of comprehensive income?
A. Rs33,795,500 B. Rs3,379,500
C. Rs3,379,550 D. Rs33,795,000

Question#9
Following is the summarized trial balance of an entity at its year end 31 December 2018
Debit Credit
Bank loan 160,000
Interest on bank loan 8,000
The bank loan was acquired on 1 April 2018. The principal amount is repayable in five equal annual
instalments on 31 March each year. Interest is payable at 10% per annum on six monthly basis and is
recorded at the time of payment. What adjusting entry is required to record interest payable as at 31
December 2018?
A. Dr Interest expense Rs. 12,000 Cr Interest B. Dr Interest expense Rs. 8,000 Cr Interest
payable Rs. 12,000 payable Rs. 8,000

Prepared by Maaz Imtiaz


75
C. Dr Interest expense Rs. 4,000 Cr Bank loan D. Dr Interest expense Rs. 4,000 , Cr Interest
Rs. 4,000 payable Rs. 4,000

Question#10
Which one of the following is not a characteristic of service organization?
A. There is no line item for the cost of goods B. The funds of service companies are
sold in the income statement of service usually tied up towards accounts
companies. receivable
C. A large percentage of assets comprise D. A large percentage of assets comprise
receivable inventory

Question#11
A business has provided following extracts from trial balance as at 30 June 2019;
Admin expenses = 15 Million Debit
Electricity expense of Rs. 0.15 million is outstanding. No adjustment for bill payable has been
recorded. Electricity expense paid during the year have already been included in administration
expenses.
Included in the administration expenses in trial balance advance rent is Rs. 0.12 million.
What is the amount of administration expenses to be shown in statement of comprehensive income?
A. Rs15.0 Million B. Rs15.27 Million
C. Rs15.15 Million D. Rs15.03 Million

Question#12
Following is summarized trial balance of an entity for year ended 30 June 2017
Rs in 000 Debit Rs in 000 Credit
Plant & machinery – cost 6,520
Plant & mach. – Acc. Dep. as at 1 July 2016 2,323
On 1 March 2017, entity paid an advance of Rs. 330,000 for purchase of a machine and debited it to
plant and machinery. The machine was delivered on 1 September 2017.
Entity depreciates its fixed assets from the month of addition. Depreciation is to be charged on
written-down value (WDV) as follows:
Plant and machinery 15%
What is the amount of depreciation to be charged to statement of profit or loss?
A. Rs386,700 B. Rs387,700
C. Rs486,700 D. Rs487,500

Question#13
Following is summarized trial balance of an entity for year ended 30 June 2017
Rs in 000 Debit Rs in 000 Credit
Bank loan 7,500
Interest expenses 825
168,928 168,928
Bank loan was received on 1 July 2016. Interest payable for the month of June 2017 has been credited
to the loan account.
What is the amount of interest payable to be shown in statement of financial position?
A. Rs68,750 B. Rs68,755

Prepared by Maaz Imtiaz


76
C. Rs68,700 D. Rs68,075

Question#14
Universal shoes (US) has prepared a trial for the year ended 31 December 2017:
Rs in 000 Debit Rs in 000 Credit
Miscellaneous income 855
Miscellaneous income includes Rs. 415,000 received against an annual maintenance contract expiring
on 30 April 2018.
What is the amount of Miscellaneous income to be shown in statement of financial position as
unearned income?
A. Rs138,300 B. Rs138,333
C. Rs138,330 D. Rs285,000

Question#15
Following is an extract from the trial balance of Hamdani Enterprises (HE) for the year ended 31
December 2019:
Rs Debit Rs Credit
Stock in trade 31-12-2018 4,500
Cost of sales 24,780
While carrying out the physical inventory count at year-end, following matters were identified:
 Goods costing Rs. 1,100 were slightly defective. These can be sold for Rs. 1,280 after incurring
a cost of Rs. 250.
 Goods costing Rs. 710 purchased on credit were returned to a supplier on 28 December 2018
but the return was not recorded in the books.
Calculate the “Inventory” amount that would be presented in statement of financial position of HE as
at 31 December 2018.
A. Rs2,690 B. Rs2,510
C. Rs3,720 D. Rs2,560

Question#16
Following is an extract from the trial balance of Hamdani Enterprises (HE) for the year ended 31
December 2019:
Rs Debit Rs Credit
Advances 450
Property, plant and equipment – Cost 12,500
Accumulated depreciation 4,630
Additional information:
 A machine costing Rs. 450 was received on 1 October 2019 against 100% advance payment.
The advance has not yet been adjusted due to non-receipt of the invoice.
 HE depreciates property, plant & equipment at 15% per annum on reducing balance method.
Calculate “property, plant and equipment” that would be presented in statement of financial position
of HE as at 31 December 2019.
A. Rs6,125 B. Rs7,826
C. RS7,123 D. Rs7,911

Prepared by Maaz Imtiaz


77
Question#17
A business has a bank balance of Rs4 800. It pays for material invoiced at Rs3 000 less trade discount
of 30% and settlement discount of 10%. A cheque for Rs450 is received from a trade receivable. What
is the bank balance after these transactions?
A. Rs3,500 B. Rs3,450
C. Rs3,360 D. Rs2,250

Question#18
Draft accounts included a loan due for payment in 20 months’ time, as part of the trade payables.
Which effect will the necessary adjustment have?
A. Reduce net current assets B. No effect on net current assets
C. Increase in net current assets D. Increase in net assets

Question#19
The following debit balance appears on a trial balance at 31 December 2019, after the preparation of
the company’s annual accounts.
Stationery Rs80,000
What is the stationery item?
A. stationery inventory in hand at 31 B. an accrual for stationery at 31 December
December 2019 2019
C. the annual stationery charge for 2019 D. an amount due to the company’s
stationery supplier

Question#20
Following is an extract from the trial balance of Hamdani Enterprises (HE) for the year ended 31
December 2019:
Rs Debit Rs Credit
Other income 1,500,000
Additional information
On 1 August 2019, MT received an amount of Rs. 180,000 as 50% advance against a maintenance
contract covering the period from 1 September 2019 to 31 May 2020 and was credited to other
income. The balance amount would be paid on completion of the contract.
What is the amount of unearned income to be shown in statement of financial position?
A. Rs160,000 B. Rs20,000
C. Rs360,000 D. Rs180,000

Question#21
A business has provided following extracts from trial balance as at 31 December 2018;
Rs Debit Rs Credit
Receivables 100,000
Provision for doubtful debts – 1.1.18 4,000
Bad debts 3,000
Additional information
A customer included in receivables in trial balance above owing Rs. 10,000 gone bankrupt and is
unable to pay anything.
It has been decided to calculate a provision of 5% for the year.
What is the amount of closing balance of provision for doubtful debts account?

Prepared by Maaz Imtiaz


78
A. Rs4,320 B. Rs4,850
C. Rs4,950 D. Rs4,500

Question#22
A business has provided following extracts from trial balance as at 31 December 2018;
Rs Debit Rs Credit
Receivables 100,000
Provision for doubtful debts – 1.1.18 4,000
Bad debts 3,000
Additional information
A customer included in receivables in trial balance above owing Rs. 10,000 gone bankrupt and is
unable to pay anything.
It has been decided to calculate a provision of 5% for the year.
What is the amount of bad and doubtful debts expense in statement of comprehensive income?
A. Rs21,500 B. Rs22,800
C. Rs22,750 D. Rs22,930

Question#23
A business has provided following extracts from trial balance as at 31 December 2018;
Rs Debit Rs Credit
Receivables 100,000
Provision for doubtful debts – 1.1.18 4,000
Bad debts 3,000
Additional information
A customer included in receivables in trial balance above owing Rs. 10,000 gone bankrupt and is
unable to pay anything.
It has been decided to calculate a provision of 5% for the year.
What is the number of receivables to be shown in statement of financial position?
A. Rs82,690 B. Rs82,700
C. Rs82,500 D. Rs82,560

Question#24
Following is summarized trial balance of an entity for year ended 30 June 2017
Rs in 000 Debit Rs in 000 Credit
Plant & machinery – cost 6,520
Plant & mach. – Acc. Dep. as at 1 July 2016 2,323
On 1 March 2017, entity paid an advance of Rs. 330,000 for purchase of a machine and debited it to
plant and machinery. The machine was delivered on 1 September 2017.
Entity depreciates its fixed assets from the month of addition. Depreciation is to be charged on
written-down value (WDV) as follows:
Plant and machinery 15%
What is the amount of Plant & machinery to be shown in statement of financial position?
A. Rs3,490,600 B. Rs3,520,900
C. Rs3,562,000 D. Rs3,480,300

Prepared by Maaz Imtiaz


79
Question#25
The table shows extract from a trial balance.
Debit (Rs) Credit
(Rs)
Prepayments 4 620
accruals 8 125
bank balances 14 920 3 612
trade payables 18 148
loan (10 years) 15 000
Which total for current liabilities should be disclosed in the balance sheet?
A. Rs44 885 B. Rs29 885
C. Rs26 380 D. Rs26 273

Question#26
A business starts trading on 1 May 2020. The table shows rent paid on the premises.
Date Period Amount
2 May 2020 1 May – 30 Jun Rs1 000
3 Jul 2020 1 Jul – 30 Sep Rs1 500
2 Oct 2020 1 Oct – 31 Dec Rs1 500
4 Jan 2021 1 Jan – 31 Mar Rs1 560
1 Apr 2021 1 Apr – 30 Jun Rs1 560
Which amount should be shown in the accounts for the year ended 30 April 2021 for rent prepayment?
A. Rs1 000 B. Rs520
C. Rs1 560 D. Rs1 040

Question#27
The table shows information from the books of a business at 30 April 2012.

Invoiced Rs
Credit sales invoiced during financial year 79 000
Goods sent to customers on 28 April 2012 and invoiced 4 May 2012 6 100
Goods sent to customers during April 2012 on sale or return basis but not sold by 30 April
2012 8 300
What is the amount to be credited to the Trading section of income statement as sales for the year
ended 30 April 2012?
A. Rs93 400 B. Rs85 300
C. Rs85 100 D. Rs76 800

Question#28
The sales included in the trial balance of a company amounting to Rs880 000. This balance is
made up asfollows:
Cash sales 215 000
Credit Sales 536 000
Goods sent on sale or return awaiting acceptance by the consignees 45 000
Goods sent on sale or return for which customers have accepted 84 000
invoices
Total 880 000

Prepared by Maaz Imtiaz


80
The sales to be recorded in the Trading section of income statement are
A. Rs880 000 B. Rs835 000
C. Rs796 000 D. Rs751 000

Question#29
The following debit balance appears on a trial balance after preparing the manufacturing
account for the year.
loose tools Rs18 000
What is this item?
A. inventory of loose tools B. a prepayment for loose tools
C. the annual charge for loose tools D. A liability for loose tools

Question#30
Which one of the statement is correct?
A. carriage outward is a debit B. carriage inward is a credit
C. sales return is a credit D. purchases return is a debit

Question#31
Following is an extract from the trial balance of Talib Enterprises (TE) for the year ended 31 December
2019:
Rs Debit Rs Credit
Prepayments and advances 1,340
Accrual and other payables 1,460
Miscellaneous income 1,010
Additional information:
 On 1 October 2019, 60% advance received for an annual maintenance contract of Rs. 510 was
credited to miscellaneous income. Remaining amount would be received at the end of the
contract. Services are rendered evenly throughout the contract period.
 Maintenance services for Rs. 200 were rendered in December 2019 but income has been
recorded in January 2020 on receipt of the amount.
Calculate “Prepayments and advances” that would be presented in statement of financial position of
TE as at 31 December 2019.
A. Rs1,470 B. Rs1,340
C. Rs1,540 D. Rs1,460

Question#32
Following is an extract from the trial balance of Talib Enterprises (TE) for the year ended 31 December
2019:
Rs Debit Rs Credit
Prepayments and advances 1,340
Accrual and other payables 1,460
Miscellaneous income 1,010
Additional information:
 On 1 October 2019, 60% advance received for an annual maintenance contract of Rs. 510 was
credited to miscellaneous income. Remaining amount would be received at the end of the

Prepared by Maaz Imtiaz


81
contract. Services are rendered evenly throughout the contract period.
 Maintenance services for Rs. 200 were rendered in December 2019 but income has been
recorded in January 2020 on receipt of the amount.
Calculate “Accrual and other payables” that would be presented in statement of financial position of
TE as at 31 December 2019
A. Rs1,450 B. Rs1,524
C. Rs1,765 D. Rs1,613

Question#33
Following is an extract from the trial balance of Talib Enterprises (TE) for the year ended 31 December
2019:
Rs Debit Rs Credit
Prepayments and advances 1,340
Accrual and other payables 1,460
Miscellaneous income 1,010
Additional information:
 On 1 October 2019, 60% advance received for an annual maintenance contract of Rs. 510 was
credited to miscellaneous income. Remaining amount would be received at the end of the
contract. Services are rendered evenly throughout the contract period.
 Maintenance services for Rs. 200 were rendered in December 2019 but income has been
recorded in January 2020 on receipt of the amount.
Calculate “Miscellaneous income” that would be presented in statement of comprehensive income of
TE for the year ended 31 December 2017.
A. Rs1,010 B. Rs1,057
C. Rs857 D. Rs1,127

Question#34
Following is an extract from the trial balance of Talib Enterprises (TE) for the year ended 31 December
2019:
Rs Debit Rs Credit
Financial charges 810
12% long term loan 5,500
Additional information:
Interest on the loan is paid in arrears on 1 April and 1 October each year. Interest accrued for the
quarter ended 31 December 2019 has been credited to loan account.
Calculate “Financial charges” that would be presented in statement of comprehensive income of TE for
the year ended 31 December 2019.
A. Rs5,500 B. Rs6,310
C. Rs660 D. Rs810

Question#35
Following is an extract from the trial balance of Talib Enterprises (TE) for the year ended 31 December
2019:
Rs Debit Rs Credit
Financial charges 810
12% long term loan 5,500

Prepared by Maaz Imtiaz


82
Additional information:
Interest on the loan is paid in arrears on 1 April and 1 October each year. Interest accrued for the
quarter ended 31 December 2019 has been credited to loan account.
Calculate “Long term loan”that would be presented in statement of comprehensive income of TE for
the year ended 31 December 2019.
A. Rs5,340 B. Rs6,310
C. Rs5,420 D. Rs6,450

Question#36
The following is an extract from the trial balance of Gaba Enterprises (GE) as at 31 December 2019:
Rs Debit Rs Credit
Revenue 1,300,000
Inventory 1 January 2019 58,000
Purchases 580,000
Carriage inwards 5,000
Carriage outwards 7,000
Administrative expenses 410,000
Distribution costs 200,000
Return inwards 1,500
Inventory valuation at 31 December 2019 was Rs. 70,000
Calculate “Administrative expenses” that would be presented in statement of comprehensive income
of GE for the year ended 31 December 2019.
A. Rs485,000 B. Rs410,000
C. Rs610,000 D. Rs480,000

Question#37
The following is an extract from the trial balance of Gaba Enterprises (GE) as at 31 December 2019:
Rs Debit Rs Credit
Revenue 1,300,000
Inventory 1 January 2019 58,000
Purchases 580,000
Carriage inwards 5,000
Carriage outwards 7,000
Administrative expenses 410,000
Distribution costs 200,000
Return inwards 1,500
Inventory valuation at 31 December 2019 was Rs. 70,000
Calculate “Distribution costs”that would be presented in statement of comprehensive income of GE
for the year ended 31 December 2019.
A. Rs207,000 B. Rs200,000
C. Rs193,000 D. Rs202,000

Question#38
The following is an extract from the trial balance of Gaba Enterprises (GE) as at 31 December 2019:
Rs Debit Rs Credit
Revenue 1,300,000

Prepared by Maaz Imtiaz


83
Inventory 1 January 2019 58,000
Purchases 580,000
Carriage inwards 5,000
Carriage outwards 7,000
Administrative expenses 410,000
Distribution costs 200,000
Return inwards 1,500
Inventory valuation at 31 December 2019 was Rs. 70,000
Calculate “Cost of sales”that would be presented in statement of comprehensive income of GE for the
year ended 31 December 2019.
A. Rs713,000 B. Rs703,000
C. Rs708,000 D. Rs573,000

Question#39
The following is an extract from the trial balance of Gaba Enterprises (GE) as at 31 December 2019:
Rs Debit Rs Credit
Revenue 1,300,000
Inventory 1 January 2019 58,000
Purchases 580,000
Carriage inwards 5,000
Carriage outwards 7,000
Administrative expenses 410,000
Distribution costs 200,000
Return inwards 1,500
Inventory valuation at 31 December 2019 was Rs. 70,000
Calculate “Gross profit”that would be presented in statement of comprehensive income of GE for the
year ended 31 December 2019.
A. Rs735,350 B. Rs725,560
C. Rs725,500 D. Rs725,900

Question#40
A business has the following assets and liabilities.
Rs
short-term investment 6 000
loan interest owing 1 500
loan repayable within one year 12 000
deposits from customers for orders 4 500
trade payables 27 000
trade receivables 39 000
pre-payments 3 500
What is the amount of net current assets?
A. Rs15 500 B. Rs8 000
C. Rs4 500 D. Rs3 500

Question#41
The table shows information from a business at 30 November 2008.

Prepared by Maaz Imtiaz


84
Rs
Credit sales invoiced during financial year 80 000
Goods dispatched to customers in November 2008 and invoiced in December 2008 5000
Goods included in sales for November 2008 on a sale or return basis, but only
sold in
December 2008
- at invoice price 10 000
- at cost price 8 000
Which amount will appear in the trading section of income statement as sales for the year ended
30November 2008?
A. Rs85 000 B. Rs83 000
C. Rs77 000 D. Rs75 000

Question#42
The directors of a company provide the following information.
Rs
bank overdraft 1 200
equipment 12 000
non-current loan 8 000
petty cash 900
inventories 2500
trade payables 3 000
trade receivables 2 000
What is the amount of the net current assets?
A. Rs5 200 B. Rs3 600
C. Rs2 400 D. Rs1 200

Question#43
A business has extracted the following information from its books of account at31 December 2014,
its firstyear of trading.

Rs000
carriage inwards 12
carriage 15
outwards
closing inventory 86
purchases 286
returns inwards 10
returns 2
outwards
revenue 524

What is the gross profit for the year ended 31 December 2014?
A. Rs328 000 B. Rs320 000
C. Rs304 000 D. Rs301 000

Question#44

Prepared by Maaz Imtiaz


85
Anoosha commenced business on 1 January 2015. For the year ended 31 December 2015, the
following information is available.

Rs
Drawings 53 500
profit for the year 62 700
Revenue 1500 000
Expenses 875 000
What was the cost of sales for the year?
A. Rs687 000 B. Rs678 000
C. Rs571 500 D. Rs562 300

Question#45
A company pays or receives the following amounts on the last day of its financial year.
Rs
deposit paid to a supplier 6 500
rental income received in advance 8 000
loan repayment 3 000
payment for last month’s sales commission 900
Which of these amounts will be included as other receivables in the statement of financial
position?
A. Rs18 400 B. Rs17 500
C. Rs14 500 D. Rs6 500

Question#46
Which are examples of the accounting equation?
1. capital + assets = liabilities
2. capital = assets + liabilities
3. capital = assets – liabilities
A. 3 only B. 2 and 3
C. 1 only D. 1 and 3

Question#47
The following information is extracted from the statement of financial position of a business at 31
December 2016.
Rs
bank loan (repayable 16 200
2025)
other payables 1 880
bank overdraft 11 600
capital 20 710
drawings 19 100
inventory 14 610
other receivables 1 420
trade payables 14 110
trade receivables 9 050
What is the value of the net current liabilities?

Prepared by Maaz Imtiaz


86
A. Rs20 320 B. Rs18 710
C. Rs2 510 D. Rs1 590

Question#48
Why does a trader account for accrued income?
A. so that profit is not understated B. so that profit is not overstated
C. so that current liabilities are not D. so that current liabilities are not
understated overstated

Question#49
A trader sends his staff on a training course costing Rs100 per person. 10 staff attended in April and 4
in May. Half the total cost had to be paid at the start of April and the balance at the end of May.
Which entry for training was made in the statement of financial position on 30 April?
A. Rs700 prepayment B. Rs600 prepayment
C. Rs400 accrual D. Rs300 accrual

Question#50
An item can be converted easily into cash.
In which section of the statement of financial position would this item appear?
A. non-current assets B. current liabilities
C. current assets D. capital

Prepared by Maaz Imtiaz


87
Chapter 10 Bank Reconciliations
Question#1
Debit balance of Rs.5,000 as per bank statement means:
A. Rs.5,000 receivable from business by the B. Rs.5,000 payable to business by the bank
bank
C. Rs.5,000 withdrawn from the bank by D. Rs.5,000 deposited by the business
business during the year during the month

Question#2

At 31 December 2019, a customer’s bank statement shows that his bank account is overdrawn by
Rs10 136. At that date, cheques drawn on his account, but not yet presented to the bank, totalled
Rs4 998 and cheques paid into his account, but not yet credited by the bank, totalled Rs5 896. His
bank statement shows that interest of Rs181 has been charged, but this has not yet been entered
in the cash book.
What is the correct bank balance to be shown in the balance sheet at 31 December 2019?
A. Rs11 034 overdrawn B. Rs10 853 overdrawn
C. Rs9 238 overdrawn D. Rs9 057 overdrawn

Question#3
The table shows extracts from business bank reconciliation.
Rs
Cash book balance in hand at 31 December 2,075
Balance as per bank statement at 31 December 2,250
Bank charges per bank statement not entered in cash book 150
Outstanding cheques not presented at the year-end 325
What is the bank balance to be shown in the financial statements?
A. Rs1 925 B. Rs1 600
C. Rs2 225 D. Rs2 075

Question#4
In the cash book of a company the bank account showed a credit balance of Rs5 000. There were
unpresented cheques amounting to Rs1 500. The bank statement showed bank charges of Rs700 not in
the cash book.
What is the balance on the bank statement?
A. Rs5 800 credit B. Rs4 200 credit
C. Rs4 200 debit D. Rs3 300 debit

Question#5
Maaz Enterprises is in process of reconciling its cash book with banks statement. Which of the
following item require entry in cash book?
A. Cheques presented by suppliers after the B. Cheque of another account erroneously
date of bank statement credited by bank
C. Deposits credited by the bank after the D. Bank service charges
date of the bank statement

Prepared by Maaz Imtiaz


88
Question#6
At the end of 31 March 2020, balance as per cash book of Hashim is Rs.10,400 (Dr) which did not agree
with the balance as per the bank statement. On investigation following information was identified;
 A standing order of Rs.450 was paid by the bank did not appear in the cash book.
 Dividend received directly in the bank was Rs.80
 Bank credited interest for the quarter Rs.350; it was included in cash book as Rs.530
 A customer cheque deposited in the bank Rs.140 was dishonoured
What is the corrected balance as per cash book?
A. 9,710 Dr B. 9,580 Dr
C. Rs.10,170 Dr D. Rs.10,170 Cr

Question#7
The following bank reconciliation statement has been prepared by an accountant:
Bank reconciliation statement as at June 30, 2019 Rs
Balance as per bank statement (Cr) 46.600
Add: Outstanding cheques 12,800
Less: Uncleared lodgements 13,900
Balance as per cash book (Dr) 45,500

What should be the correct balance as per cash book?


A. Rs45,500 Dr B. Rs45,500 Cr
C. Rs47,700 Dr D. Rs47,700 Cr

Question#8
Balance of bank account as per cash book is Rs.34,000 (Dr) while balance as per bank statement is
Rs.32,000 (Cr). Difference is explained as Uncleared lodgements of Rs.2,000 not included in the bank
statement.
What is the amount of bank balance to be reported in Statement of financial Position?
A. Rs.32,000 Overdraft B. Rs.32,000 Cash at bank
C. Rs.34,000 Overdraft D. Rs.34,000 Cash at bank

Question#9
A business is in process of preparing its bank reconciliation statement. The balance of cash book did
not agree with the balance in bank statement. The following information is available:
 Balance as per cash book before comparing bank statement Rs. 12,000 (Dr)
 Outstanding Cheques Rs.1,660
 Outstanding lodgements Rs.1,250
 Bank charges Rs.70
 Bank interest income Rs.90
What is the balance as per bank statement?
A. Rs12,430 Dr B. Rs12,430 Cr
C. Rs12,230 Dr D. Rs12,230 Cr

Prepared by Maaz Imtiaz


89
Question#10
If we found that the receipt side of the cash book has been under-casted, then in preparing bank
reconciliation statement, it should :
A. Deducted from balance as per bank B. Added in balance as per cash book
statement
C. Added in balance as per bank statement D. Deducted from balance as per cash book

Question#11
The balance at bank in X’s cash book at 30 April is Rs12 460 debit. However, a cheque for Rs14 470
received from Y and a cheque for Rs1 740 paid to Z appears in the cash book but not on the bank
statement. Bank charges of Rs4 500 have not been entered in the cash book.
What is the balance shown on the bank statement at 30 April?
A. Rs20 690 debit B. Rs20 690 credit
C. Rs4 770 debit D. Rs4 770 credit

Question#12
A bank reconciliation statement shows a credit balance of Rs400 in the Cash Book and a balance in
hand of Rs100 in the bank statement.
The bank reconciliation statement includes unpresented cheques of Rs700 in addition to cheques
banked and not yet credited in the bank statement.
What is the total of cheques banked and not yet credited?
A. Rs1 200 B. Rs1 000
C. Rs400 D. Rs200

Question#13
When preparing a bank reconciliation statement the following information is available.
Rs
Bank balance shown by the cash 20 000 debit
bookUnpresented cheques 2 500
Uncleared bankings 1 400
Standing order shown on the bank statement (not in the cash 300
book)
What is the balance on the bank statement?
A. Rs21 400 B. Rs20 800
C. Rs19 200 D. Rs18 600

Question#14
Which one of the following statements is correct?
A. Credit balance as per cash book means an B. Debit balance as per cash book means a
asset bank overdraft
C. Debit balance as per bank statement D. Credit balance as per bank statement
means a bank overdraft means a bank overdraft

Question#15
Following information has been collected from the books of Kamran as at 31 January 2019: Balance as
per cash book Rs.14,000 (Dr)
On scrutiny of bank statement it was found:

Prepared by Maaz Imtiaz


90
 Unpresented Cheques Rs.2,500
 Uncredited lodgements Rs.1,500
 Bank charges Rs.190
 Bank debits Kamran for bank interest Rs.120 instead of Rs.140. No amount was recorded in cash
book of Kamran
Further it was found that:
 Receipt of Rs.1,350 was recorded on credit side of cash book
 Payment of Rs.1,100 was recorded on debit side of cash book
What is the corrected cash book balance of Kamran?
A. Rs14,170 Dr B. Rs14,170 Cr
C. Rs15,990 Dr D. Rs15,990 Cr

Question#16
The following information pertains to bank reconciliation:
 The bank balance in the cash book before taking the items below into account was Rs.9,770
overdrawn
 Bank charges of Rs.700 on the bank statement have not been entered in the cash book
 The bank has credited the account in error with Rs.650 which belongs to another customer
 Cheque payments totalling Rs.4,395 have been entered in the cash book but have not been
presented by payment
 Cheques totalling Rs.6,400 have been correctly entered on the debit side of the cash book but
have not been paid in at the bank
What was the balance as shown by the bank statement?
A. Rs11,955 Overdrawn B. Rs11,825 Overdrawn
C. Rs11,925 Overdrawn D. Rs11,885 Overdrawn

Question#17
A check written by the company for Rs167 is incorrectly recorded by a company as Rs176. On the bank
reconciliation, the Rs9 error should be
A. Deducted from the balance per bank. B. Added to the balance per bank.
C. Deducted from the balance per books. D. Added to the balance per books.

Question#18
Which one of the following require deduction from cash book balance while preparing bank
reconciliation statement?
A. Bank debits interest Rs. 2,600 instead of B. Unpresented cheques not yet paid by
Rs. 6,200, the correct amount has been bank
included in cash book
C. Standing orders paid by the bank not yet D. Direct deposit by a customer into bank
entered in cash book but not yet entered in cash book

Question#19
For which of the following errors should the appropriate amount be added to the balance per bank on
a bank reconciliation?
A. Check for Rs75 recorded by the company B. A returned Rs300 check recorded by the
as Rs57. bank as Rs30.

Prepared by Maaz Imtiaz


91
C. Deposit of Rs600 recorded by the bank as D. Check for Rs63 recorded by the company
Rs60. as Rs36.

Question#20
Which of the following would be added to the balance per bank on a bank reconciliation?
A. Service charges. B. Notes collected by the bank.
C. Deposits in transit D. Outstanding checks.

Question#21
Which of the following items on a bank reconciliation would require an adjusting entry on the
company’s books?
A. A deposit in transit B. A bank service charge.
C. Outstanding checks D. An error by the bank.

Question#22
Balance as per bank statement of Asim was Rs.11,800 credit as on April 30, 2018, which was not in
agreement with the balance as per cash book. On investigation the following items were detected:
 Cheques issued and paid by the bank for Rs.4,500 but recorded in the cash book as Rs.500
 Bank service charges not entered in the cash book Rs.620
 Outstanding lodgements Rs.1,4300
 Bank has erroneously debited a cheque of Rs.950 to Asim actually the cheque was issued by Asif.
 Unpresented cheques Rs.1,300
What should be the balance as per cash book before adjustments?
A. Rs. 17,470 Credit B. Rs. 17,470 Debit
C. Rs. 12,850 Credit D. Rs. 12,850 Debit

Question#23
The bank column of a cash book showed a credit balance of Rs.8,500. There were unpresented
cheques amounting to Rs.2,600. The bank statement showed bank charges, Rs.920, which were not
recorded in the cash book. What is the balance on the bank statement?
A. Rs9,420 Normal B. Rs9,420 Overdraft
C. Rs6,820 Normal D. Rs6,820 Overdraft

Question#24
Balance as per bank statement was Rs.1,500 in debit. Comparison of bank statement with cash book
revealed that cheques of Rs.4,000 paid in per the cash book but not yet on the bank statement and
cheques of Rs.850 paid out but not yet appeared on the bank statement. In addition the bank
statement shows direct deposit of Rs.750 by a customer but it is not recorded in cash book.
What is the balance as per cash book after adjustments?
A. Rs.1,650 Positive Balance B. Rs. 1,650 Overdraft
C. Rs.1,880 Positive Balance D. Rs. 1,880 Overdraft

Question#25
When preparing a bank reconciliation statement the following information is available
 Balance as per cash book Rs.26,000 (Dr)
 Outstanding cheques Rs.1,800

Prepared by Maaz Imtiaz


92
 uncleared lodgements 1,500
 Standing order shown on the bank statement (not appearing in a cash book) Rs.150
 Dividend directly deposited in the bank (not appearing in the cash book) Rs.40
What is the balance as per bank statement?
A. Rs26,190 Overdraft B. Rs26,190 Normal
C. Rs25,096 Overdraft D. Rs26,096 Normal

Question#26
The main purpose of preparing a bank reconciliation statement is?
A. To know the bank balance B. To know the balance of bank statement
C. To identify causes of difference between D. To correct the cash book
cash book and bank statement

Question#27
Favorable balance means?
A. Credit balance in the cash book B. Debit balance in cash book
C. Credit balance in Bank statement D. Both B and C

Question#28
Bank reconciliation statement is?
A. A sub-division of journal B. A separate statement
C. Part of the cash book D. Part of bank statement

Question#29
Uncollected checks are also known as?
A. Uncleared checks B. Outstation checks
C. Outstanding checks D. Both A and B

Question#30
Bank reconciliation is not a?
A. Procedure to provide cash book B. Ledger account
adjustments
C. Memorandum statement D. Reconcile records

Question#31
Determine the balance as per bank statement using the following information:
 Balance as per cash book Rs. 48,100
 Cheques received and deposited into the bank, but not yet credited in the bank statement Rs.
4,950
 Unpresented cheques Rs. 6,900
 Credit transfers appearing in the bank statement but not entered in the cash book Rs. 3,600
A. Rs53,650 B. Rs52,920
C. Rs53,750 D. Rs52,960

Question#32

Prepared by Maaz Imtiaz


93
Following information has been extracted from the records of Lightning Store (LS), as at 30 June 2016:
Outstanding cheques amount to Rs. 3,986,400 and include:
 A cheque of Rs. 55,000 issued to a supplier bearing an incorrect payee's name. The cheque was
returned and recorded on 15 July 2016.
 A cheque issued to a supplier for Rs. 86,000 (included in above amount correctly) was recorded
in bank book as Rs. 62,000.
 A cheque dated 20 December 2015 for Rs. 4,830 issued for repair of a car was misplaced. The
repair charges were paid in cash and the misplacement of the cheque was not recorded.
Balance as per bank statement (overdraft) Rs. 1,580,650 .
Assuming that cash book balance has been corrected and adjusted, what amount of bank balance shall
be reported in statement of financial position?
A. Rs5,588,290 Positive balance B. Rs5,588,290 Overdraft
C. Rs5,507,220 Positive balance D. Rs5,507,220 Overdraft

Question#33
Following information has been extracted from the records of Lightning Store (LS), as at 30 June 2016:
Un-cleared cheques amount to Rs. 6,580,000 and include:
 A cheque of Rs. 378,000 received from a customer was returned by the bank as amount in
words was not in conformity with the amount in figures. The return was not recorded and the
cheque was sent to the customer for replacement.
 A cheque of Rs. 79,000 received from a customer in settlement of an invoice availing payment
discount of 3%. The collection was recorded by EG at gross amount of invoice.
Balance as per bank statement (overdraft) Rs. 1,580,650 .

Assuming that cash book balance has been corrected and adjusted, what amount of bank balance shall
be reported in statement of financial position?
A. Rs5,426,650 Positive balance B. Rs5,426,650 Overdraft
C. Rs4,621,350 Positive balance D. Rs4,621,350 Overdraft

Question#34
Which one of the following is not missing of cash book?
A. Interest credited in bank statement B. Bank charges debited in bank statement
C. Outstation check D. Mistakes in cash book

Question#35
When cash is deposited into bank then the following account would be debited in the company
accounts?
A. Bank account B. Cash account
C. Overdraft account D. None

Question#36
While reconciling the bank statement with the cash/bank book of Alpha company for the year ended
December 31, 2015, you noted the following:
Balance as per bank statement at December 31, 2015, overdrawn Rs. 804,986
Cheques drawn but not presented till December 31, 2015, Rs. 387,884
A time-barred cheque of Rs. 5,550 was replaced with a new cheque on December 30, 2015, and

Prepared by Maaz Imtiaz


94
entered in the cash/bank book without the previous cheque being cancelled / reversed. Both the
cheques are included in above unpresented cheques.
Assuming that cash book balance has been corrected and adjusted, what amount of bank balance shall
be reported in statement of financial position?
A. Rs1,187,870 Credit B. Rs1,187,870 Debit
C. Rs1,297,890 Credit D. Rs1,297,890 Debit

Question#37
While reconciling the bank statement with the cash/bank book of Alpha company for the year ended
December 31, 2015, you noted the following:
Balance as per bank statement at December 31, 2015, overdrawn Rs. 804,986
Cheques drawn but not presented till December 31, 2015, Rs. 387,884
The above unpresented cheques include a cheque of Rs. 36,000 issued to supplier that is time barred
as of January 3, 2016.
Assuming that cash book balance has been corrected and adjusted, what amount of bank balance shall
be reported in statement of financial position?
A. Rs1,292,240 Credit B. Rs1,292,240 Debit
C. Rs1,192,870 Credit D. Rs1,192,870 Debit

Question#38
The balance on the debit side of the bank column of cash book indicates?
A. Cash at bank B. The total amount overdraft in the bank
C. The total amount has drawn from the D. None
bank

Question#39
When bank statement shows a debit balance, it means?
A. Unfavorable balance as per bank book B. Unfavorable balance as per cash book
C. Overdraft balance as per cash book D. None

Question#40
The check which is deposited into bank but not cleared at the end of a particular year is called?
A. Omitted check B. Unpresented check
C. Uncredited check D. Dishonored check

Question#41
The table shows details relating to a company’s banking transactions at 31 December.
Rs
Balance at bank per bank 22 650
statementUn-cleared 3 110
lodgments 6 290
Un-presented cheques 650
Bank credit recorded twice by bank in error
Which balance for cash at bank should appear in the Balance Sheet at 31 December?
A. Rs26 480 B. Rs25 180

Prepared by Maaz Imtiaz


95
C. Rs20 120 D. Rs18 820

Question#42
A draft Balance Sheet shows a bank balance of Rs1 400. The following information is now available.
Rs
cheques issued but not yet cleared by the bank 150
bank charges not in the cash book 45
lodgments in the cash book but not on the bank statement 220
What is the figure shown on the Bank Statement?
A. Rs1 515 B. Rs1 425
C. Rs1 355 D. Rs1 285

Question#43
The following items are recorded in the cash book of a business but not yet recorded in its bank
statement:
Rs
Cheques drawn 3000
Amounts banked 250
The cash book shows a bank overdraft of Rs2600. What is the balance on the bank statement?
A. Rs400 overdraft B. Rs400 in hand
C. Rs150 overdraft D. Rs150 in hand

Question#44

At the financial year end of a business the following information is available.


Rs
debit balance on the bank statement 1 000
Unpresented cheques 300
lodgements not yet credited by the bank 600
bank charges and interest charged not yet entered in the cash book 150
What is the current balance in the cash book?
A. Rs550 debit B. Rs550 credit
C. Rs400 debit D. Rs400 credit

Question#45
A bank reconciliation statement has been prepared by an inexperienced book-keeper.

Rs
bank statement balance (68 100)
(overdrawn)cheques received 141 200
not paid in 209 300
(41 800)
cheques paid to suppliers, not yet 167 500)
presented cash book balance
(overdrawn)
What is the correct bank balance according to the cash book?
A. Rs167 500 B. Rs167 500 overdrawn

Prepared by Maaz Imtiaz


96
C. Rs31 300 D. Rs31 300 overdrawn

Question#46
When dealing with Bank reconciliation statement while using missing method the credit side of cash
book corresponds to?
A. Credit side of Bank statement B. Debit side of cash book
C. Debit side of Bank statement D. None

Question#47
In cash book bank charges recorded on which side?
A. Debit side B. Credit side
C. Both A and B D. None

Question#48
Credit balance as per pass book is?
A. Favorable balance B. Unfavorable balance
C. Both A and B D. None

Question#49
An amount of Rs. 1,000 is debited twice in the bank statement. When overdraft as per the cash book is
the starting point?
A. Rs. 1,000 will be added B. Rs. 1,000 will be deducted
C. Rs. 2,000 will be added D. Rs. 2,000 will be deducted

Question#50
If any amount is directly deposited into the bank then?
A. Cash book will show less balance & bank B. Bank book will show double balance
book will show more
C. Cash book will show double balance D. Cash book will show more balance & bank
book will show less

Prepared by Maaz Imtiaz


97
Chapter 1 ACCOUNTING FUNDAMENTALS

Answer Key
1. C 2. D 3. B 4. D
5. C 6. A 7. B 8. D
9. A 10. B 11. D 12. A
13. B 14. C 15. D 16. A
17. B 18. A 19. C 20. D
21. B 22. DA A 23. DA C 24. DA D
25. DA B 26. DA D 27. A 28. A
29. A 30. A 31. C 32. DA A
33. DA C 34. B 35. B 36. A
37. D 38. D 39. A 40. A
41. B 42. D 43. A 44. B
45. C 46. DA A 47. DA C 48. DA B
49. DA C 50. DA A

DA = Detailed answers are provided below.


22) Extention in factory building + overhaul in machinery = 2.0+1.9 = 3.9Mn
23) Repairs to factory building = Rs 0.6 Mn
24) Profit – Revenue expenditure = 10.6 – 0.6 = 10Mn (extension and overhaul are capital
expenditure)
25) Opening cost of factory building + Construction cost during the year = 18 + 5 = 23Mn
26) Cost of factory vehicle – opening = 9.9Mn ( repairs to factory vehicle is revenue expenditure )
32) Accounting equation : Asset = Liabilities + Equity
3.2 = 1.2 + Equity Equity = 3.2 – 1.2 = 2Mn
33) 8,990 + 7,050 + 8,950 = 24,990 ( current assets have ecomic benefits of not more than one
year )
46) 2.6 + 0.6 – 0.9 = 2.3Mn
47) 3.5 + 0.5 – 2.9 = 1.1Mn
48) 950,000 – 120,000 = 830,000
49) 3.1 + 1.2 – 0.6 = 3.7Mn
50) 5.9 + 4.5 – 8.3 = 2.1Mn

98
Chapter 2 Books of prime entry
Answer key
1 D 15 A 29 DA A 43 DA A
2 B 16 A 30 DA A 44 A
3 C 17 D 31 DA B 45 DA C
4 B 18 B 32 DA C 46 A
5 C 19 C 33 DA C 47 B
6 D 20 D 34 DA B 48 C
7 C 21 DA C 35 DA C 49 DA B
8 A 22 DA C 36 DA C 50 D
9 B 23 DA A 37 DA B
10 D 24 D 38 D
11 C 25 DA C 39 D
12 B 26 D 40 A
13 B 27 C 41 DA A
14 B 28 A 42 DA A

DA = Detailed answer provided below .


Detailed Answer
21) Gross Profit = Sales-Cost of sales = 120,000-48,000 = 72,000
Net profit = Gross profit – expenses = 72,000-43,000 = 29,000
22) Capital employed is long term capital + long term liablities OR Fixed assets + current assets –
current liablities .
23) Rs5,000,000 is capital receipt as fixed asset is sold
Rs 500,000 is revenue receipt as cash received from services
25) Plates and cutlery are fixed assets hence capital expenditure . Foods and drinks are are
revenue expenditure.
29) Capital = Total assets – Total liablities
= 14,000,000 + 1,250,000 + 1,875,000 – (850,000+ 375,000) = 15,910,000
30) Provision at year end = 13,400 – 600 = 12,800*5% = Rs640 .
31) Current year provision = 41,000*2.5% = 1,025
Adjustment for provision = Rs1,075 - 1,025 = Rs 50 decrease
32) Capital = Total assets – total liablities = 44000 + 1500 + 1500 – (10000+3300) = Rs 34,800
33) Credit sales are entered in the sales day books; sales returns are entered in sales returns day
book (not deducted from sales day book total); cash sales and cash received from customers
are entered in cash receipt book
34) Total sales = Rs. 24,000+ (Rs. 20,000x0.97) = Rs. 43,400
35) (28,000-5000)*0.98=22,540
36) 900,000-(110,000*2)=680,000 Due to posting on wrong side; for correction double amount is
to be credited.
37) 170,000+18,000+5,000+25,000- 190,000=28,000
41) Gross profit =Exp. + Profit= 15,000+7,000=22,000 Revenue=G.P+CoS= 22,000+80,000=102,000
42) Plates and cutlery are fixed assets hence capital expenditure . Foods and drinks are are
revenue expenditure.

99
43) Decrease in the provision for doubtful debts increases the gross profit means income
statement is credited.
45) Opening balance provision – 70,000*3% = 2,100
Closing balance provision = 73,000*3% = 2,190
Increase in provision = 90
This provision is Debit in income statement as an expense.
49) Rs. 55,000+8000+47,000-75,000= Rs. 35,000

100
Chapter3 Ledgers and Trial balance
Answer Key
1. A 2. B 3. B 4. B
5. D 6. B 7. A 8. A
9. D 10. A 11. D 12. D
13. D 14. D 15. C 16. A
17. C 18. D 19. A 20. D
21. A 22. B 23. C 24. D
25. C 26. C 27. D 28. D
29. A 30. B 31. C 32. C
33. D 34. C 35. DA B 36. DA A
37. B 38. DA B 39. DA D 40. DA D
41. B 42. A 43. DA B 44. A
45. A 46. D 47. A 48. C
49. A 50. D

DA = Detailed answers are provided below.


35)
Accounts receivables
Particulars Rs. Particulars Rs.
b/d 46,652 Cash 165,900
Sales 245,000 Discount allowed 1,200
c/d 124,552
291,652 291,652

36)
Accounts Payable
Particulars Rs. Particulars Rs.
Bank 195,000 b/d 66,450
Discount received 28,900 Purchases 205,600
Receivable (contra) 12,700
c/d 35,450
272,050 272,050

38)
Accounts Payable

101
Particulars Rs. Particulars Rs.
Bank 100,600 Opening (b/d) 138,000
Discounts Recieved 5,500 Purchases 55,600

c/d 87,500
193,600 193,600
39)
Cash a/c
Particulars Rs. Particulars Rs.
b/d 250,000 Payables 58,000
Receivables 92,000 c/d 284,000
342,000 342,000

40) 159,763 + 46,325 + 69,578 + 29,453 + 2,548 = 307,667

41)
Accounts Recievable
Particulars Rs. Particulars Rs.
Opening (Bal.) 558,440 Bank 563,890
Sales 452,880 Discount allowed 28,900
Payable(contra) 67,580

1,011,320 1,011,320

43)
Building a/c
Particulars Rs.Mn Particulars Rs.Mn
b/d 90 Disposal 19
Additions 40 c/d 111
130 130

102
Chapter 4 Accruals and Prepayments
Answer Key
1. A 2. B 3. C 4. A
5. A 6. A 7. DA A 8. C
9. C 10. D 11. C 12. A
13. C 14. C 15. C 16. C
17. A 18. A 19. B 20. B
21. B 22. B 23. DA D 24. DA A
25. DA D 26. C 27. DA B 28. DA B
29. D 30. B 31. DA B 32. C
33. B 34. D 35. D 36. B
37. C 38. A 39. D 40. C
41. B 42. DA C 43. A 44. C
45. A 46. DA A 47. DA D 48. DA C
49. DA C 50. D

DA = Detailed answers are provided below.


7) Rs120,000 + 500,000 – 550,000 = Rs70,000
23)
Rent
Particulars Rs. Particulars Rs.
b/d(Accrued) 28,000
For the year 45,000 Cash (bal.) 38,000
c/d 35,000
73,000 73,000

24) 120,000 - (30,000+30,000+30,000+30,000/3) = 20,000 prepaid


25) 19,000- 7,250 = 11,750
27) 400,000 – 265,000 – 50,000 = 85,000
28)
Rent
Particulars Rs. Particulars Rs.
b/d(Accrued) 19,550 b/d(advance) 35,700
Profit or loss (bal.) 135,050 Cash recieved 130,500
c/d(advance) 40,500 c/d(accrued) 28,900
195,100 195,100

103
30) Step 1 : 690,000/12 *10 = 575,000 ( rent of 10 months)
Step 2 : 690,000/110 * 100 = 627,273 To calculate rent of previous year
Step 3 : 627,273/12 * 2 = 104,546 Rent of Jan and Feb
Step 4 : 575,000 + 104,546 = 679,546
31) (1,200,000*8/12)+(1,500,000*4/12) = 1,300,000
42) 1,200,000*0.08 = 96,000 Annual interest => 96,000*2/4 = 48,000 Interest of two quarter
46)
Rs Rs
Amount received 38,400
Less : prepayment for 2021 (5,400)
33,000
Received in 2021 34,000
Recievable for 2020 34,500/3 11,500
44,500
47) Paid Rs75,000 on 30 September 2020
This covers July/August 2020 (2x25,000) Rs50,000
So there was an accrual for September 2020
For September 2020 25,000
Paid 1.1.2021 (3x Rs25,000) 75,000
Paid 1.4.2021 (3x Rs25,000) 75,000
Paid 1.7.2021 (3x Rs25,000) 75,000
Accrued for July/Aug (2 x 25,000) 50,000
Expense for year 300,000

48) Prepaid 9,800 – 6,000 = 3,800


Paid for 3 months = 6000*3/12 = 1,500
Expense for 2021 5,300
49)
Rs Rs
Balance at 31.12.2020 216,500
Less : Service contract (27,000)
189,500
Service contract 27,000
Apportion to 2010 (5/36 x 27,000) 3,750 3,750
Prepaid 23,250
Expense 193,250

104
Chapter 5 Bad and doubtful debts
Answer Key
1. D 2. A 3. DA B 4. C
5. DA B 6. DA A 7. DA C 8. A
9. DA B 10. B 11. A 12. A
13. A 14. D 15. C 16. C
17. C 18. B 19. A 20. A
21. DA D 22. D 23. A 24. DA D
25. D 26. DA D 27. C 28. B
29. C 30. D 31. DA A 32. DA D
33. A 34. C 35. DA D 36. C
37. DA B 38. DA C 39. DA A 40. A
41. C 42. C 43. DA B 44. DA B
45. C 46. DA D 47. C 48. DA A
49. D 50. C

DA = Detailed answers are provided below.


3) Increase in provision (Rs10,700,000  Rs5,900,000) 4,800,000
Bad debts written off (Rs750,000  Rs50,000) 700,000
Amount to be charged in Income statement 5,500,000

5) 2,300,000*4% = 92,000  92,000 – 12,000(op. allowance) = 80,000

6) (142,400-2,000) * 3% = 4,212

7)
Rs
Total bad debts written off 1 500
Increase in provision for doubtful debts (Rs2 500-Rs1 000) 1 500
Less Bad debts recovered (100)
Net amount to be charged to Income statement 2 900

8) Closing receivables (Rs700,000  Rs30,000) 670,000


Closing allowances (Rs670,000*0.08) (53,600)
Carrying amount of receivables 616,400

9) Closing allowance = (Rs. 80,000 – 10,000) x4% = Rs. 2,800


(Rs. 10,000 x 40%) = Rs. 4,000
6,800

21)
Rs Rs
Bad debts 800

105
Bad debts recovery (200)
Provision for doubtful debts required [(Rs17 800 - Rs800)* 340
2%]
Provision for doubtful debts available 580
Decrease in provision (240)
Total bad and doubtful debts 360

23) Expense = Bad debts + increase in allowance = Rs22,000+50,000= Rs72,000

24)
Rs
Provision for doubtful debts required [(Rs69 200 - Rs480)* 3 436
5%]
Existing provision for doubtful debts 2 600
Increase in provision 836

Increase in provision for a doubtful debt involves the


following entry.
Dr Income statement Rs836
Cr Provision for doubtful debts Rs836

26) Expense = Rs. 90,000+ (Rs. 180,000-160,000) = Rs. 110,000


Profit will decrease by Rs. 110,000

29)
Rs
Provision for doubtful debts required [(Rs10 620 -Rs260)* 5%] 518
Provision for doubtful debts brought forward 460
Increase in existing provision 58

31)
Rs
Provision for doubtful debts required (Rs125 400 *5%) 6 270
Provision for doubtful debts at 1 January 2013 (1 800)
Doubtful expense for the year 4 470

32) Closing balance of allowance


Rs. 93,000 – 1,800 bad debts = Rs. 91,200 x 5% = Rs. 4,560
Opening balance of allowance Rs. 4,560 x 100/120 = Rs. 3,800

35)
Rs
Total of current assets on 30 September 28000
Increase in inventories (Rs2 000 × 80%) 1 600
Decrease in cash (Rs2 000 × 80% × 95%) (1 520)
Decrease in trade receivables due to bad debts (400)
Total of current assets on 2 October 27 680

106
37) Closing allowance = Rs. 95,000x6% = Rs. 5,700
Charge for Statement of profit or loss Rs.
Decrease in allowance (300)
Irrecoverable debts 4,000
Bad debts recovered (1,700)
Profit or loss 2,000

38) 22,000 – (42,000-35,000) = 15,000

39) Rs. 1,200,000 – 1% discount as payment made within 10 days = Rs. 1,188,000

43) Rs. 900,000 – 120,000 = Rs. 780,000 – 1% discount as payment made within 10 days = Rs.
772,200
44) Allowance = (54,200 x 0%) + (32,500 x 5%) + (12,080 x 49%) = Rs. 7,544.2
Decrease in allowance = Rs. 7,250 – 7,544.2 opening = Rs. (294.2)

46) Rs. 3,000,000 x 5% x 4% = Rs. 4,000 x 33.33/133.33 = Rs. 1,500 Increase in allowance would
increase expense, resulting in reduction in profit.

48) 26,000+(22,000-30,000) = 18,000

107
Chapter 6 Depreciation
Answer Key
1. B 2. B 3. D 4. D
5. B 6. B 7. A 8. B
9. D 10. A 11. B 12. DA D
13. D 14. B 15. DA C 16. A
17. DA D 18. C 19. A 20. B
21. D 22. C 23. C 24. DA B
25. B 26. A 27. D 28. C
29. DA D 30. A 31. C 32. B
33. A 34. C 35. D 36. DA A
37. DA B 38. DA D 39. DA B 40. A
41. DA B 42. DA B 43. C 44. DA A
45. DA B 46. B 47. C 48. DA D
49. DA B 50. D

DA = Detailed answers are provided below.


12) 1500,000*0.15*3 = 675,000Dep. 1,500,000-675,000=825,000 B.V
825,000 – 450,000 = 375,000

15) Depreciable amount = Rate 150,000 *100 = 0.15


Cost * useful life 200,000*5

17) 1,500,000 – 300,000 = 1,200,000 1,200,000*2/4 = 600,000


1,500,000 – 600,000 = 900,000

24) Cost 1.5


Dep (0.375) @25%
B.V 1.125
Dep (0.28125) @25%
B.V 0.84375
Dep (0.2109375) @25%
B.V 0.6328125 0.6328125*10^6 = Rs632,813
Concept : To convert Rs in Mn to Rs multiply it by 10 to the power 6 .

25) Cost 200,000


Dep (30,000) @15%
B.V 170,000
Dep (25,500) @15%
B.V 144,500 B.V = Sale value = 144,500

26) Machinery cost + delivery cost + modification cost = Total cost


46,000,000+800,000+2,900,000 = 49,700,000 = 49.7 Mn

108
29)
Rs
Freehold land 4,500,000
Architect fees 150,000
Site preparation 200,000
Materials 2,250,000
Direct labor costs 1,120,000
Legal fees 50,000
Total 8,270,000

36) 65,000,000*1,600/38,000 = 2,736,842

37) Sum of digits = 5+4+3+2+1 = 15


Cost 900,000
Dep (270,000) (900,000-90,000)*5/15
B.V 630,000
Dep (216,000) (900,000-90,000)*4/15
B.V 414,000

38) Six months depreciation is calculated


Rs
Land(undepreciated) 2,000,000
Building(10,000 – (10,000/25 × 6/12)) 9,800,000
Air conditioning system (4,000 – (3,500/10 × 6/12)) 3,825,000
15,625,000

39) Rs225,000
Disposal A/c
Particulars Rs. Particulars Rs.
Cost 300,000 Cash 90,000
Profit 15,000 Accumulated depreciation(bal.) 225,000

315,000 315,000

41) Cost of premises:


Rs in 000
Cost 400,000
Adaption 12,000
Legal fees 2,500
414,500

109
42) 200,000/5 = 40,000

44) 15,000
Disposal A/c
Particulars Rs. Particulars Rs.
Cost 1,200,000 Cash 840,000
Profit (bal.) 230,000 Accumulated depreciation 590,000

1,430,000 1,430,000

Rs in 000
45) List price of machine 82,000
Less: trade discount 8% (6,560)
75,440
Less: cash discount 4% (3,017.6)
Import duty 1,500
Delivery fees 2,050
Electrical installation costs 9,500
Pre-production testing 4,900
90,372.4
46) 430,000
Disposal A/c
Particulars Rs. Particulars Rs.
Cost 600,000 Cash(Bal.) 430,000
Profit 70,000 Accumulated depreciation 240,000

670,000 670,000

48) (100,500-40,900+20,500-7,000)*0.25 = 18,275


49)

Rs
Cost of machine sold 270,000
Depreciation for first year (270,000 ´ 20%) (54,000)
216,000
Depreciation for second year (216,000 ´ (43,200)
20%)
Book value of machine sold 172,800
Sales price of machine sold 120,000
Loss on machine disposal 52,800

110
Chapter 7 IAS-2 Inventories
Answer Key
1. A 2. D 3. B 4. C
5. DA A 6. B 7. A 8. DA A
9. DA D 10. B 11. C 12. A
13. B 14. A 15. B 16. DA D
17. DA A 18. A 19. C 20. DA D
21. A 22. B 23. A 24. DA C
25. B 26. B 27. DA C 28. B
29. A 30. B&D 31. C 32. B
33. B 34. D 35. D 36. DA C
37. B 38. B 39. D 40. B
41. B&D 42. A 43. D 44. C
45. DA C 46. A 47. C 48. DA B
49. B&D 50. D

DA = Detailed answers are provided below.

5) 1,020+170+60+265+115+1600 = 3,230

9) (15*350)+(12*360)+(11*250) = 12,320

10) 30,000+7,500+29,000 = 8,500


140,400-8,500 = 131,900

16) 11,000,000+850,000+200,000+250,000 = 12,300,000

17) Gross profit = Rs. 1,200,000 – 550,000 = Rs. 650,000


Net profit = Rs. 650,000 – 60,000 – 60,000 = Rs. 530,000

20) Cost Rs. 158,000 is lower NRV Rs. 253,000 (i.e. Rs. 260,000 – 7,000) is higher
Replacement cost Rs. 255,000 is irrelevant

24) 10 September (Rs. 40,000 + 15,000)/ (400+100 units) = Rs. 110 per unit

10 September (Rs. 55,000 + 60,000)/ (500+300 units) = Rs. 143.75 per unit

27) Debit Cost of Sales Rs. 1,568,000 & Credit Inventory Rs. 1,568,000
Rs. 1,880,000 + 80,000 = Rs. 1,960,000 x 100/125 = Rs. 1,568,000

36) Cost is Rs26,300


NRV = Rs47,500 – Rs8,900 – Rs2,000 = Rs36,600

111
45) Sales – costs of sales = gross profit
Rs. 4,600,000 – (Rs. 1,600,000+2,650,000-900,000) = Rs. 1,250,000

48) Cost per unit = [(36,000x3.9) +(41,000x4.5)]/ (36,000+41,000) =Rs. 4.22


Cost of goods sold = 60,000 x Rs. 4.22 = Rs. 253,200

112
Chapter 8 Accounting for Manufacturing
Answer Key
1. C 2. D 3. A 4. C
5. C 6. D 7. C 8. B
9. A&B 10. B&C 11. D 12. B
13. B 14. C 15. C 16. B
17. DA A 18. A 19. B 20. A
21. B 22. DA C 23. DA A 24. DA D
25. DA A 26. A 27. D 28. DA B
29. DA B 30. DA B 31. DA D 32. DA B
33. DA D 34. B 35. B 36. DA A
37. A 38. B 39. A 40. DA D
41. C 42. A 43. B 44. DA A
45. DA A 46. A 47. A 48. B
49. C 50. A

DA = Detailed answers are provided below.


17) 810*790 = 639,900

22) 11,990 hours x Rs. 510 per hour = Rs. 6,114,900

23) Absorbed Overheads = Rs. 510 x (11,990 x 103%) = Rs. 6,298,347

24) Actual – Absorbed = Rs. 5,648,900 – 6,298,347 = Rs.649,447 overabsorbed

25) Prime cost = Direct materials + Direct labour


(30,000+2,500)+29,000 = 61,500

28) 8,000+4,000+1,000+4,500 = 17,500

29) 550,000 + 190,000 = 740,000

30) 550,000 + 190,000 + 35000 – 35,000 = 740,000

32) Rs. 370,000 + 55,000 + 22,500 (royalty) = Rs. 447,500

33) Rs. 17,000 indirect + Rs. 25,000 x 80% Depreciation + Rs. 12,500 rent and utilities = Rs. 49,500

36) 290,000+120,000-18,000 = 392,000

40) (42,000+115,000+75,000)+(25,000*50%)+(15,000*50%) = 252,000

44) 76,000+(425,000-27,000) = 474,000

113
45) 76,000+(425,000-27,000)-110,000 = 364,000

114
Chapter 9 Preparation of Financial Statements
Answer Key
1. B 2. B 3. B 4. B
5. DA A 6. D 7. A 8. DA D
9. DA D 10. D 11. DA D 12. DA A
13. DA A 14. DA B 15. DA C 16. C
17. C 18. C 19. A 20. B
21. DA D 22. DA A 23. DA C 24. DA D
25. B 26. D 27. DA C 28. DA B
29. A 30. A 31. DA C 32. DA D
33. DA B 34. D 35. DA A 36. B
37. DA A 38. DA D 39. DA C 40. D
41. DA D 42. DA D 43. DA C 44. DA D
45. D 46. A 47. DA C 48. A
49. D 50. C

DA = Detailed answers are provided below.

5) Drawings = Rs. 300,000 – (15000x12) = Rs. 120,000

8) Sales(36,260,000 – 240,000 – 475,000) 35,545,000


Less: Sales return 1,750,000
33,795,000

9) 160,000*10%*3/12 = 4,000

11) = Rs. 15.0 + 0.15 -0.12 = Rs. 15.03 million

12) Depreciation charge = [6,520 – 330-2,323] x 10% = Rs. 386,700

13) 825*1/12 = 68,750

14) 415,000 * 4/12 = 138,333

15) Inventory as given Rs. 4,500 – write down Rs. 70 – return to supplier Rs. 710 = Rs. 3,720

16) At cost Rs. 13,800 + 550 = Rs. 14,350


Accumulated depreciation Rs. 5,290 + 1,197 = Rs. 5,827
Net Rs. 7,123

17)
Rs
Bank balance at start 4 800

115
Less Material cost paid * (1 890)
Add Cheque received from trade receivables 450
Bank balance at end 3 360

*List price of material 3 000


Less Trade discount (Rs3 000 30%) (900)
2 100
Less Settlement (cash) discount (Rs2 100 10%) (210)
Amount actually paid for material 1 890

20) Total contract price 180,000 + 180,000 = 360,000 x 4/9 months = 160,000 less advance 180,000
= 20,000 unearned

21) 100,000 – 10,000 = 90,000


90,000 * 5% = 4,500

22) Baddebts 3000 + 10,000= 13,000


Increase in provision 4,000 + 4,500(as above) = 8,500
21,500

23) Receivables as per trial 100,000


Bad debts (3,000+10,000) (13,000)
87,000
Closing provision (4,500)
82,500

24) 6,520,000-330,000-2,323,000-386,700 = 3,480,300

27) Rs79 000 + Rs6 100 = Rs85 100

28) Total sales = Rs215 000 + Rs536 000 + Rs84 000 = Rs835 000

31) Rs. 1,340 + 200 income receivable = Rs. 1,540

32) Rs. 1,460 + 153 unearned income = Rs. 1,613


Unearned income Rs. 510 x 60% x 3/6 months = Rs. 153

33) Rs. 1,010 – 153 unearned + 200 accrued = Rs.1,057

35) Rs. 5,500 x 100 / 103 = Rs. 5,340

37) Rs. 200,000 + Rs. 7,000 Carriage outwards = Rs. 207,000

38) Cost of sales = 58,000+580,000+5,000-70,000 = 573,000

39) Revenue 1,300,000 – Return Inwards 1500 = 1,298,500


Cost of sales = (573,000)
= 725,500

116
40)
Current Assets Rs Rs
Short-term investment 6 000
Trade receivables 39 000
Pre-payments 3 500 48 500
Current Liabilities
Loan interest owing 1 500
Loan repayable within one year 12 000
Deposits from customers for orders 4 500
Trade payables 27 000 (45 000)
Net Current Assets 3 500

41) Rs80000 + Rs5 000  Rs10 000 = Rs75 000

42)
Rs
Current Assets (Rs2 500 + Rs2 000 + Rs900) 5 400
Current Liabilities (Rs1 200 + Rs3 000) (4 200)
Net Current Assets 1 200

43)
Rs00 Rs000
0
Revenue 524
Returns inwards 10 514
Cost of sales
Purchases 286
Add Carriage inwards 12
Less Returns outwards (2)
Less Closing inventory (86) (210)
Gross profit for the year 304

44)
Rs
Sales 1 500 000
Cost of Sales (Rs1 500 000 - Rs937 700) (562 300)
Gross profit (Rs62 700 + Rs875 000) 937 700
Expenses (875 000)
Profit for the year 62 700

47)
Current Liabilities Rs Rs
Other payables 1 880
Bank overdraft 11 600
Trade payables 14 110 27 590
Current Assets
Inventory 14 610
Other receivables 1 420
Trade receivables 9 050 (25 080)

117
Net current liabilities 2 510

118
Chapter 10 Bank Reconciliations
Answer Key
1. A 2. DA C 3. DA A 4. DA C
5. D 6. DA A 7. DA C 8. D
9. DA B 10. B 11. DA C 12. DA D
13. DA B 14. C 15. DA A 16. DA B
17. D 18. C 19. C 20. C
21. B 22. DA B 23. DA D 24. DA A
25. DA B 26. C 27. D 28. B
29. D 30. B 31. A 32. DA D
33. DA C 34. C 35. A 36. DA A
37. DA C 38. A 39. A 40. C
41. DA D 42. DA D 43. DA D 44. DA B
45. DA C 46. C 47. B 48. A
49. A 50. A

DA = Detailed answers are provided below.

2)
Rs
Overdraft balance as per adjusted cash book (balancing figure) 9,238
Add Uncreditedcheques 5,896
15,124
Less Unpresented cheques (4,998)
Overdraft balance as per bank statement 10,136

3)
Rs
Balance as per adjusted cash book [balancing figure) 1,925
Add Unpresented cheques 325
Balance as per bank statement 2,250

4)
Rs
Bank overdraft as per cash book (Rs5 000 + Rs700) 5,700
Less Unpresented cheques (1,500)
Balance as per bank statement 4,200

6) 10,400-450+80-180-140 = 9,710

7) 46,600-12,800+13,900 = 47,700

119
9) (12,000-70+90)+1,660-1,250 = 12,430

11)
Balance as per adjusted cash book (Dr) (Rs12 460 – Rs4 500) 7 960
Add Unpresented cheques 1 740
Less Uncreditedcheques (14 470)
Balance as per bank statement (Dr) 4 770

12)
Balance as per cash book (Cr) 400
Add Uncreditedcheques (balancing figure) 200
Less Unpresented cheques (700
Balance as per bank statement (Cr) 100

13)
Bank Reconciliation Statement Rs
Balanc as per adjusted Cash book (Dr) 19700
e
Add Unpresented cheques 2 500
22 200
Less Uncleared deposits (1 400)
Balanc as per bank statement 20 800
e

15) Cash Book: Rs. 14,000 (unadjusted) – 190 bank charges - 140 interest charged + 1,350 Receipts
x 2 times – 1,100 payments x 2 times = Rs. 14,170 (Adjusted)

16) Cash Book: Rs. (9,770) (unadjusted) – 700 bank charges = Rs. (10,470) (Adjusted) Bank
Reconciliation Statement: Rs. (10,470) (Adjusted Cash Book) + 4,395 Unpresented – 6,400
Uncredited + 650 error = Rs. (11,825) (Bank Statement)

22) Bank Reconciliation Statement: Rs. 11,800 (Bank Statement) – 1,300 Unpresented + 1,400
Uncredited + 950 error = Rs. 12,850 (Adjusted Cash Book)
Cash Book: Rs. ? (unadjusted) – 4,000 unrecorded payments – 620 bank charges = Rs. 12,850
(Adjusted)
Unadjusted Balance calculated from above equation = Rs. 17,470 (Positive balance)

23) Cash Book: Rs. (8,500) (unadjusted) – 920 Bank charges = Rs. (9,420) (Adjusted) Bank
Reconciliation Statement: Rs. (9,420) (Adjusted Cash Book) + 2,600 Unpresented = Rs. (6,820)
(Bank Statement)

24) Bank Reconciliation Statement: Rs. (1,500) (Bank Statement) – 850 Unpresented + 4,000
Uncredited = Rs. 1,650 (Adjusted Cash Book)

25) (26,000-150+40)+1,800-1,500 = 26,190

120
31) Cash Book: Rs. 48,100 (unadjusted) + 3,600 Credit transfers = Rs. 51,700 (Adjusted) Bank
Reconciliation Statement: Rs. 52,500 (Adjusted Cash Book) + 6,900 Unpresented – 4,950
Uncredited = Rs. 53,650 (Bank Statement)

32) Unpresented cheques = Rs. 3,986,400 – 55,000 cancelled – 4,830 cancelled = Rs. 3,926,570
Bank reconciliation statement: Rs. (1,580,650) bank statement – 3,926,570unpresented = Rs.
(5,507,220) Adjusted cash book

33) Uncredited items = Rs. 6,580,000 – 378,000 dishonoured cheque = Rs. 6,202,000
Bank reconciliation statement: Rs. (1,580,650) bank statement + 6,202,000Uncredited = Rs.
4,621,350 Adjusted cash book

36) Unpresented cheques = Rs. 387,884 – 5,550 cancelled = Rs. 382,334


Bank reconciliation statement: Rs. (804,986) – 382,884 unpresented = Rs. (1,187,870) Adjusted
cash book

37) Bank reconciliation statement: Rs. (804,986) – 387,884 unpresented = Rs. (1,192,870) Adjusted
cash book

41)
Balance as per bank statement 22,650
Add Uncleared lodgments 3,110
Less Unpresented cheques (6,290)
Bank credit recorded twice by bank (650)
Balance as per adjusted cash book 18,820

42)
Bank Reconciliation Statement Rs
Balance as per adjusted cash book (Rs1 400 Rs45) 1 355
Add Un-presented cheques 150
Less Un-credited cheques (220
Balance as per bank statement 1 285

43)

Overdraft balance as per cash book 2 600


(Cr)
Add Amounts banked but not credited 250
Less Cheques drawn but not presented (3 000)
Balance as per bank statement (Cr) 150

121
44)
Balance as per bank statement 1,000
Add Unpresented cheques 300
Less lodgements not yet credited by the bank (600
Balance as per adjusted cash book (Credit) 700
Adjusted Cash Book
Rs Rs
Balance c/d (Bank reconciliation) 700 Balance b/f (balancing figure) 550
Bank charges and interest 150
700 700

45) Bank Reconciliation Statement

Overdraft balance as per bank statement (Dr) 68 100


Add cheques paid to suppliers, not yet presented (Un-presented 41 800
Cheques)
Less cheques received not paid in (Un-credited cheques) (141 200)
Balance as per adjusted cash book (Dr) 31,300

122

You might also like